CardioNerds Decipher The Guidelines Logo

2022 AHA / ACC / HFSA Guideline for The Management of Heart Failure

Welcome to the CardioNerds Decipher the Guidelines 2022 AHA / ACC / HFSA Guideline for The Management of Heart Failure edition! 

The Decipher the Guidelines using high-impact, board-style, clinical vignette based questions to highlight core concepts relevant to your practice. We will do so by releasing several short bite-sized Pods with one question per episode.

This particular series is built around the 2022 AHA / ACC / HFSA Guideline for The Management of Heart Failure and was developed by the CardioNerds and created in collaboration with the American Heart Association and the Heart Failure Society of America. It was created by 30 trainees and 16 faculty experts, with mentorship from Dr. Anu Lala, Dr. Robert Mentz, and Dr. Nancy Sweitzer. We thank Dr. Judy Bezanson and Dr. Elliott Antman for their guidance.

The cases discussed in this podcast series are fictional and designed to highlight key teaching points.

The following question refers to Section 13 of the 2022 AHA/ACC/HFSA Guideline for the Management of Heart Failure.

The question is asked by Western Michigan University medical student and CardioNerds Intern Shivani Reddy, answered first by Mayo Clinic Cardiology Fellow and CardioNerds Academy Faculty Dr. Dinu Balanescu, and then by expert faculty Dr. Harriette Van Spall.

Dr. Van Spall is an Associate Professor of Medicine, cardiologist, and Director of E-Health at McMaster University. Dr Van Spall is a Canadian Institutes of Health Research-funded clinical trialist and researcher with a focus on heart failure, health services, and health disparities.

The question is asked by Keck School of Medicine USC medical student & former CardioNerds Intern Hirsh Elhence, answered first by Vanderbilt Cardiology Fellow and CardioNerds Academy Faculty Dr. Breana Hansen, and then by expert faculty Dr. Javed Butler.

Dr. Butler is an advanced heart failure and transplant cardiologist, President of the Baylor Scott and White Research Institute, Senior Vice President for the Baylor Scott and White Health, and Distinguished Professor of Medicine at the University of Mississippi

The following question refers to Section 8.5 of the 2022 AHA/ACC/HFSA Guideline for the Management of Heart Failure.

The question is asked by Western Michigan University medical student & CardioNerds Intern Shivani Reddy, answered first by University of Southern California cardiology fellow and CardioNerds FIT Trialist Dr. Michael Francke, and then by expert faculty Dr. Shashank Sinha.

Dr. Sinha is an Assistant Professor of Medical Education at the University of Virginia School of Medicine and an advanced heart failure, MCS, and transplant cardiologist at Inova Fairfax Medical Campus. He currently serves as both the Director of the Cardiac Intensive Care Unit and Cardiovascular Critical Care Research Program at Inova Fairfax. He is also a Steering Committee member for the multicenter Cardiogenic Shock Working Group and Critical Care Cardiology Trials Network and an Associate Editor for the Journal of Cardiac Failure, the official Journal of the Heart Failure Society of America.

The following question refers to Section 7.8 of the 2022 AHA/ACC/HFSA Guideline for the Management of Heart Failure.

The question is asked by Stony Brook University Hospital medicine resident and CardioNerds Intern Dr. Chelsea Tweneboah, answered first by Mayo Clinic Cardiology Fellow and CardioNerds Academy Chief Dr. Teodora Donisan, and then by expert faculty Dr. Michelle Kittleson.

The following question refers to Section 7.3 of the 2022 AHA/ACC/HFSA Guideline for the Management of Heart Failure.

The question is asked by Palisades Medical Center medicine resident & CardioNerds Academy Fellow Dr. Maryam Barkhordarian, answered first by Hopkins Bayview medicine resident & CardioNerds Academy Faculty Dr. Ty Sweeny, and then by expert faculty Dr. Gregg Fonarow.

Dr. Fonarow is the Professor of Medicine and Interim Chief of UCLA’s Division of Cardiology, Director of the Ahmanson-UCLA Cardiomyopathy Center, and Co-director of UCLA’s Preventative Cardiology Program.

The following question refers to Section 7.2 of the 2022 AHA/ACC/HFSA Guideline for the Management of Heart Failure.

The question is asked by Cleveland Clinic internal medicine resident and CardioNerds Intern Akiva Rosenzveig, answered first by UPMC Harrisburg cardiology fellow and CardioNerds Academy House Faculty Leader Dr. Ahmed Ghoneem, and then by expert faculty Dr. Randall Starling.

Dr. Starling is Professor of Medicine and an advanced heart failure and transplant cardiologist at the Cleveland Clinic where he was formerly the Section Head of Heart Failure, Vice Chairman of Cardiovascular Medicine, and member of the Cleveland Clinic Board of Governors. Dr. Starling is also Past President of the Heart Failure Society of America in 2018-2019. Dr. Staring was among the earliest CardioNerds faculty guests and has since been a valuable source of mentorship and inspiration. Dr. Starling’s sponsorship and support was instrumental in the origins of the CardioNerds Clinical Trials Program.

The following question refers to Section 4.3 of the 2022 AHA/ACC/HFSA Guideline for the Management of Heart Failure.

The question is asked by Texas Tech University medical student and CardioNerds Academy Intern Dr. Adriana Mares, answered first by Rochester General Hospital cardiology fellow and Director of CardioNerds Journal Club Dr. Devesh Rai, and then by expert faculty Dr. Eldrin Lewis.

Dr. Lewis is an Advanced Heart Failure and Transplant Cardiologist, Professor of Medicine and Chief of the Division of Cardiovascular Medicine at Stanford University.

The following question refers to Sections 6.1 and 7.3 of the 2022 AHA/ACC/HFSA Guideline for the Management of Heart Failure.

The question is asked by Keck School of Medicine USC medical student & former CardioNerds Intern Hirsh Elhence, answered first by Greater Baltimore Medical Center medicine resident and CardioNerds Academy Fellow Dr. Alaa Diab, and then by expert faculty Dr. Mark Drazner.

Dr. Drazner is an advanced heart failure and transplant cardiologist, Professor of Medicine, and Clinical Chief of Cardiology at UT Southwestern. He is the past President of the Heart Failure Society of America.

The following question refers to Sections 10.2 of the 2022 AHA/ACC/HFSA Guideline for the Management of Heart Failure.

The question is asked by Western Michigan University medical student and CardioNerds Intern Shivani Reddy, answered first by Mayo Clinic Cardiology Fellow and CardioNerds Academy House Faculty Leader Dr. Dinu Balanescu, and then by expert faculty Dr. Ileana Pina.

Dr. Pina is Professor of Medicine and Quality Officer for the Cardiovascular Line at Thomas Jefferson University, Clinical Professor at Central Michigan University, and Adjunct Professor of Biostats and Epidemiology at Case Western University. She serves as Senior Fellow and Medical Officer at the Food and Drug Administration’s Center for Devices and Radiological Health.

The following question refers to Section 9.3 of the 2022 AHA/ACC/HFSA Guideline for the Management of Heart Failure.

The question is asked by Keck School of Medicine USC medical student & CardioNerds Intern Hirsh Elhence, answered first by Cedars Sinai medicine resident, soon to be Vanderbilt Cardiology Fellow, and CardioNerds Academy Faculty Dr. Breanna Hansen, and then by expert faculty Dr. Anu Lala.

Dr. Lala is an advanced heart failure and transplant cardiologist, associate professor of medicine and population health science and policy, Director of Heart Failure Research, and Program Director for the Advanced Heart Failure and Transplant fellowship training program at Mount Sinai. Dr. Lala is Deputy Editor for the Journal of Cardiac Failure. Dr. Lala has been a champion and role model for CardioNerds. She has been a PI mentor for the CardioNerds Clinical Trials Network and continues to serve in the program’s leadership. She is also a faculty mentor for this very 2022 heart failure decipher the guidelines series.

The question is asked by Western Michigan University medical student & CardioNerds Intern Shivani Reddy, answered first by University of Southern California cardiology fellow and CardioNerds FIT Trialist Dr. Michael Francke, and then by expert faculty Dr. Prateeti Khazanie.

Dr. Khazanie is an associate professor and advanced heart failure and transplant Cardiologist at the University of Colorado. Dr. Khazanie is an author on the 2022 ACC/AHA/HFSA HF Guidelines, the 2021 HFSA Universal Definition of Heart Failure, and multiple scientific statements.

The following question refers to Section 7.6 of the 2022 AHA/ACC/HFSA Guideline for the Management of Heart Failure.

The question is asked by premedical student and CardioNerds Intern Pacey Wetstein, answered first by Mayo Clinic Cardiology Fellow and CardioNerds Academy Chief Dr. Teodora Donisan, and then by expert faculty Dr. Nancy Sweitzer.

Dr. Sweitzer is Professor of Medicine, Vice Chair of Clinical Research for the Department of Medicine, and Director of Clinical Research for the Division of Cardiology at Washington University School of Medicine. She is the editor-in-chief of Circulation: Heart Failure. Dr. Sweitzer is a faculty mentor for this Decipher the HF Guidelines series.

The following question refers to Sections 7.3.2, 7.3.8, and 7.6.2 of the 2022 AHA/ACC/HFSA Guideline for the Management of Heart Failure.

The question is asked by Palisades Medical Center medicine resident & CardioNerds Intern Dr. Maryam Barkhordarian, answered first by Hopkins Bayview medicine resident & CardioNerds Academy Fellow Dr. Ty Sweeny, and then by expert faculty Dr. Robert Mentz.

Dr. Mentz is associate professor of medicine and section chief for Heart Failure at Duke University, a clinical researcher at the Duke Clinical Research Institute, and editor-in-chief of the Journal of Cardiac Failure. Dr. Mentz is a mentor for the CardioNerds Clinical Trials Network as lead principal investigator for PARAGLIDE-HF and is a series mentor for this very Decipher the Guidelines Series. For these reasons and many more, he was awarded the Master CardioNerd Award during ACC22.

The following question refers to Section 7.1 of the 2022 AHA/ACC/HFSA Guideline for the Management of Heart Failure.

The question is asked by New York Medical College medical student and CardioNerds Intern Akiva Rosenzveig, answered first by Lahey Hospital and Medical Center internal medicine resident and CardioNerds Academy House Faculty Leader Dr. Ahmed Ghoneem, and then by expert faculty Dr. Clyde Yancy.

Dr. Yancy is Professor of Medicine and Medical Social Sciences, Chief of Cardiology, and Vice Dean for Diversity and Inclusion at Northwestern University, and a member of the ACC/AHA Joint Committee on Clinical Practice Guidelines.

The following question refers to Sections 3.2, 4.1, 4.3, and 4.4 of the 2022 AHA/ACC/HFSA Guideline for the Management of Heart Failure.

The question is asked by Texas Tech University medical student and CardioNerds Academy Intern Dr. Adriana Mares, answered first by Baylor University cardiology fellow and CardioNerds FIT Trialist Dr. Shiva Patlolla, and then by expert faculty Dr. Shelley Zieroth.

Dr. Zieroth is an advanced heart failure and transplant cardiologist, Head of the Medical Heart Failure Program, the Winnipeg Regional Health Authority Cardiac Sciences Program, and an Associate Professor in the Section of Cardiology at the University of Manitoba. Dr. Zieroth is a past president of the Canadian Heart Failure Society. She is a steering committee member for PARAGLIE-HF and a PI Mentor for the CardioNerds Clinical Trials Program.

The following question refers to Section 5.1 of the 2022 AHA/ACC/HFSA Guideline for the Management of Heart Failure.

The question is asked by Keck School of Medicine USC medical student & CardioNerds Intern Hirsh Elhence, answered first by Greater Baltimore Medical Center medicine resident / Johns Hopkins MPH student and CardioNerds Academy House Chief Dr. Alaa Diab, and then by expert faculty Dr. Biykem Bozkurt.

Dr. Bozkurt is the Mary and Gordon Cain Chair, Professor of Medicine, Director of the Winters Center for Heart Failure Research, and an advanced heart failure and transplant cardiologist at Baylor College of Medicine in Houston, TX. She is former President of HFSA, former senior associate editor for Circulation, and current Editor-In-Chief of JACC Heart Failure. Dr. Bozkurt was the Vice Chair of the writing committee for the 2022 Heart Failure Guidelines.

The following question refers to Sections 11.3 of the 2022 AHA/ACC/HFSA Guideline for the Management of Heart Failure.

The question is asked by Western Michigan University medical student and CardioNerds Intern Shivani Reddy, answered first by Johns Hopkins Osler internal medicine resident and CardioNerds Academy Fellow Dr. Justin Brilliant, and then by expert faculty Dr. Harriette Van Spall.

Dr. Van Spall is Associate Professor of Medicine, cardiologist, and Director of E-Health at McMaster University. Dr Van Spall is a Canadian Institutes of Health Research-funded clinical trialist and researcher with a focus on heart failure, health services, and health disparities.

The following question refers to Section 10.1 of the 2022 AHA/ACC/HFSA Guideline for the Management of Heart Failure

The question is asked by Western Michigan University medical student and CardioNerds Intern Shivani Reddy, answered first by Boston University cardiology fellow and CardioNerds Ambassador Dr. Alex Pipilas, and then by expert faculty Dr. Ileana Pina.

Dr. Pina is Professor of Medicine and Quality Officer for the Cardiovascular Line at Thomas Jefferson University, Clinical Professor at Central Michigan University, and Adjunct Professor of Biostats and Epidemiology at Case Western University. She serves as Senior Fellow and Medical Officer at the Food and Drug Administration’s Center for Devices and Radiological Health.

The following question refers to Section 9.5 of the 2022 AHA/ACC/HFSA Guideline for the Management of Heart Failure

The question is asked by Keck School of Medicine USC medical student & CardioNerds Intern Hirsh Elhence, answered first by Duke University cardiology fellow and CardioNerds FIT Ambassador Dr. Aman Kansal, and then by expert faculty Dr. Javed Butler.

Dr. Butler is an advanced heart failure and transplant cardiologist, President of the Baylor Scott and White Research Institute, Senior Vice President for the Baylor Scott and White Health, and Distinguished Professor of Medicine at the University of Mississippi.

The following question refers to Section 9.1 of the 2022 AHA/ACC/HFSA Guideline for the Management of Heart Failure

The question is asked by Keck School of Medicine USC medical student & CardioNerds Intern Hirsh Elhence, answered first by Duke University cardiology fellow and CardioNerds FIT Ambassador Dr. Aman Kansal, and then by expert faculty Dr. Anu Lala.

Dr. Lala is an advanced heart failure and transplant cardiologist, associate professor of medicine and population health science and policy, Director of Heart Failure Research, and Program Director for the Advanced Heart Failure and Transplant fellowship training program at Mount Sinai. Dr. Lala is deputy editor for the Journal of Cardiac Failure. Dr. Lala has been a champion and role model for CardioNerds. She has been a PI mentor for the CardioNerds Clinical Trials Network and continues to serve in the program’s leadership. She is also a faculty mentor for this very 2022 heart failure decipher the guidelines series.

The following question refers to Section 9.5 of the 2022 AHA/ACC/HFSA Guideline for the Management of Heart Failure

The question is asked by Western Michigan University medical student & CardioNerds Intern Shivani Reddy, answered first by Brigham & Women’s medicine resident and Director of CardioNerds Internship Dr. Gurleen Kaur, and then by expert faculty Dr. Shashank Sinha.

Dr. Sinha is an Assistant Professor of Medical Education at the University of Virginia School of Medicine and an advanced heart failure, MCS, and transplant cardiologist at Inova Fairfax Medical Campus. He currently serves as both the Director of the Cardiac Intensive Care Unit and Cardiovascular Critical Care Research Program at Inova Fairfax. He is also a Steering Committee member for the multicenter Cardiogenic Shock Working Group and Critical Care Cardiology Trials Network and an Associate Editor for the Journal of Cardiac Failure, the official Journal of the Heart Failure Society of America.

The question is asked by Western Michigan University medical student & CardioNerds Intern Shivani Reddy, answered first by Brigham & Women’s medicine resident and Director of CardioNerds Internship Dr. Gurleen Kaur, and then by expert faculty Dr. Prateeti Khazanie.

Dr. Khazanie is an Associate Professor and Advanced Heart Failure and Transplant Cardiologist at the University of Colorado. She was an undergraduate at Duke University as a B.N. Duke Scholar. She spent two years at the NIH in the lab of Dr. Anthony Fauci and completed a dual MD-MPH program at Duke Medical School. When she started residency, she thought she was going to be an ID doctor, but she fell in love with cardiology at Stanford where she was an intern, resident, and then chief resident. She went back to Duke for her general cardiology and advanced heart failure/transplant fellowships as well as research training at the DCRI. Dr. Khazanie joined the University of Colorado in 2015 as a health services clinician researcher with a focus on improving health equity and bioethics in advanced heart failure care. She mentors medical students, residents, and fellows and is a faculty mentor for the University of Colorado Cardiology Fellows “House of Cards” mentoring group. She has research funding from the NIH/NHLBI K23, NIH Ethics Grant, and Ludeman Center for Women’s Health Research. Dr. Khazanie is an author on the 2022 ACC/AHA/HFSA HF Guidelines, the 2021 HFSA Universal Definition of Heart Failure, and multiple scientific statements.

The following question refers to Section 7.7 of the 2022 AHA/ACC/HFSA Guideline for the Management of Heart Failure

The question is asked by St. George’s University medical student and CardioNerds Intern Chelsea Tweneboah, answered first by Baylor College of Medicine Cardiology Fellow and CardioNerds Ambassador Dr. Jamal Mahar, and then by expert faculty Dr. Michelle Kittleson.

Dr. Kittleson is Director of Education in Heart Failure and Transplantation, Director of Heart Failure Research, and Professor of Medicine at the Smidt Heart Institute, Cedars-Sinai. She is Deputy Editor of the Journal of Heart and Lung Transplantation, on Guideline Writing Committees for the American College of Cardiology (ACC)/American Heart Association, is the Co Editor-in-Chief for the ACC Heart Failure Self-Assessment Program, and on the Board of Directors for the Heart Failure Society of America. Her Clinician’s Guide to the 2022 Heart Failure guidelines, published in the Journal of Cardiac Failure, are a must-read for everyone!

The question is asked by premedical student and CardioNerds Intern Pacey Wetstein, answered first by Baylor College of Medicine Cardiology Fellow and CardioNerds Ambassador Dr. Jamal Mahar, and then by expert faculty Dr. Nancy Sweitzer.

Dr. Sweitzer is Professor of Medicine, Vice Chair of Clinical Research for the Department of Medicine, and Director of Clinical Research for the Division of Cardiology at Washington University School of Medicine. She is the editor-in-chief of Circulation: Heart Failure. Dr. Sweitzer is a faculty mentor for this Decipher the HF Guidelines series.

The following question refers to Section 7.3 of the 2022 AHA/ACC/HFSA Guideline for the Management of Heart Failure

The question is asked by Palisades Medical Center medicine resident & CardioNerds Intern Dr. Maryam Barkhordarian, answered first by MedStar Washington Hospital Center cardiology hospitalist & CardioNerds Academy Graduate Dr. Luis Calderon, and then by expert faculty Dr. Gregg Fonarow.

Dr. Fonarow is the Professor of Medicine and Interim Chief of UCLA’s Division of Cardiology, Director of the Ahmanson-UCLA Cardiomyopathy Center, and Co-director of UCLA’s Preventative Cardiology Program.

The question is asked by Palisades Medical Center medicine resident & CardioNerds Intern Dr. Maryam Barkhordarian, answered first by MedStar Washington Hospital Center cardiology hospitalist & CardioNerds Academy Graduate Dr. Luis Calderon, and then by expert faculty Dr. Robert Mentz. 

Dr. Mentz is associate professor of medicine and section chief for Heart Failure at Duke University, a clinical researcher at the Duke Clinical Research Institute, and editor-in-chief of the Journal of Cardiac Failure. Dr. Mentz is a mentor for the CardioNerds Clinical Trials Network as lead principal investigator for PARAGLIDE-HF and is a series mentor for this very 2022 heart failure Decipher the Guidelines Series. For these reasons and many more, he was awarded the Master CardioNerd Award during ACC22.  

The following question refers to Section 7.4 of the 2022 AHA/ACC/HFSA Guideline for the Management of Heart Failure. The question is asked by New York Medical College medical student and CardioNerds Intern Akiva Rosenzveig, answered first by Cornell cardiology fellow and CardioNerds Ambassador Dr. Jaya Kanduri, and then by expert faculty Dr. Randall Starling.

Dr. Starling is Professor of Medicine and an advanced heart failure and transplant cardiologist at the Cleveland Clinic where he was formerly the Section Head of Heart Failure, Vice Chairman of Cardiovascular Medicine, and member of the Cleveland Clinic Board of Governors. Dr. Starling is also Past President of the Heart Failure Society of America in 2018-2019. Dr. Staring was among the earliest CardioNerds faculty guests and has since been a valuable source of mentorship and inspiration. Dr. Starling’s sponsorship and support was instrumental in the origins of the CardioNerds Clinical Trials Program.

The following question refers to Section 7.1 of the 2022 AHA/ACC/HFSA Guideline for the Management of Heart Failure. The question is asked by New York Medical College medical student and CardioNerds Intern Akiva Rosenzveig, answered first by Cornell cardiology fellow and CardioNerds Ambassador Dr. Jaya Kanduri, and then by expert faculty Dr. Clyde Yancy.

Dr. Yancy is Professor of Medicine and Medical Social Sciences, Chief of Cardiology, and Vice Dean for Diversity and Inclusion at Northwestern University, and a member of the AHA/ACC/HFSA Heart Failure Guideline Writing Committee.

The following question refers to Section 4.1 of the 2022 AHA/ACC/HFSA Guideline for the Management of Heart Failure. The question is asked by Texas Tech University medical student and CardioNerds Academy Intern Dr. Adriana Mares, answered first by Baylor University cardiology fellow and CardioNerds FIT Trialist Dr. Shiva Patlolla, and then by expert faculty Dr. Eldrin Lewis.

Dr. Lewis is an Advanced Heart Failure and Transplant Cardiologist, Professor of Medicine and Chief of the Division of Cardiovascular Medicine at Stanford University.

The following question refers to Section 3.1 of the 2022 AHA/ACC/HFSA Guideline for the Management of Heart Failure. The question is asked by Texas Tech University medical student and CardioNerds Academy Intern Dr. Adriana Mares, answered first by Rochester General Hospital cardiology fellow and Director of CardioNerds Journal Club Dr. Devesh Rai, and then by expert faculty Dr. Shelley Zieroth.

Dr. Zieroth is an advanced heart failure and transplant cardiologist, Head of the Medical Heart Failure Program, the Winnipeg Regional Health Authority Cardiac Sciences Program, and an Associate Professor in the Section of Cardiology at the University of Manitoba. Dr. Zieroth is a past president of the Canadian Heart Failure Society. She is a steering committee member for PARAGLIE-HF and a PI Mentor for the CardioNerds Clinical Trials Program.

The following question refers to Section 6.1 of the 2022 AHA/ACC/HFSA Guideline for the Management of Heart Failure. The question is asked by Keck School of Medicine USC medical student & CardioNerds Intern Hirsh Elhence, answered first by Mount Sinai Hospital cardiology fellow and CardioNerds FIT Trialist Dr. Jason Feinmanand then by expert faculty Dr. Mark Drazner.

Dr. Drazner is an advanced heart failure and transplant cardiologist, Professor of Medicine, and Clinical Chief of Cardiology at UT Southwestern. He is the President of the Heart Failure Society of America.

The following question refers to Section 2.1 of the 2022 AHA/ACC/HFSA Guideline for the Management of Heart Failure. The question is asked by Keck School of Medicine USC medical student & CardioNerds Intern Hirsh Elhence, answered first by Mount Sinai Hospital cardiology fellow and CardioNerds FIT Trialist Dr. Jason Feinman, and then by expert faculty Dr. Biykem Bozkurt.

Dr. Bozkurt is the Mary and Gordon Cain Chair, Professor of Medicine, Director of the Winters Center for Heart Failure Research, and an advanced heart failure and transplant cardiologist at Baylor College of Medicine in Houston, TX. She is former President of HFSA, former senior associate editor for Circulation, current Editor-In-Chief of JACC Heart Failure. Dr. Bozkurt was the Vice Chair of the writing committee for the 2022 Heart Failure Guidelines.

Join CardioNerds (Dr. Mark Belkin and Dr. Natalie Tapaskar) as they discuss the 2022 AHA/ACC/HFSA Guideline for the Management of Heart Failure with Writing Committee Chair Dr. Paul Heidenreich. They discuss how one gets involved with a guideline writing committee, the nuts and bolts of the guideline writing process, pitfalls and utility of the term “GDMT,” background behind inclusion of “Value Statements,” potential omissions from the document, clinical uptake of recommendations, and anticipated changes for the next iteration. Audio editing by CardioNerds academy internPace Wetstein.

This discussion is a prelude to the CardioNerds Decipher The Guidelines Series designed to enhance understanding and uptake of the 2022 AHA/ACC/HFSA Guideline for the Management of Heart Failure. We will be using high-impact, board-style, clinical vignette-based questions to highlight core concepts relevant to your practice. We will do so by releasing several short bite-sized Pods with one question per episode. Note that the cases used are hypothetical and created solely to illustrate core concepts.

Question and Answers

Palliative and supportive care has a role for patients with heart failure only in the end stages of their disease.

TRUE

FALSE

Explanation

The correct answer is False

Palliative care is patient- and family-centered care that optimizes health-related quality of life by anticipating, preventing, and treating suffering and should be integrated into the management of all stages of heart failure throughout the course of illness. The wholistic model of palliative care includes high-quality communication, estimation of prognosis, anticipatory guidance, addressing uncertainty, shared decision-making about medically reasonable treatment options, advance care planning; attention to physical, emotional, spiritual, and psychological distress; relief of suffering; and inclusion of family caregivers in patient care and attention to their needs during bereavement.

As such, for all patients with HF, palliative and supportive care—including high-quality communication, conveyance of prognosis, clarifying goals of care, shared decision-making, symptom management, and caregiver support—should be provided to improve QOL and relieve suffering (Class 1, LOE C-LD).

For conveyance of prognosis, objective risk models can be incorporated along with discussion of uncertainty since patients may overestimate survival and the benefits of specific treatments – “hope for the best, plan for the worst.”

For clarifying goals of care, the exploration of each patient’s values and concerns through shared decision-making is essential in important management decisions such as when to discontinue treatments, when to initiate palliative treatments that may hasten death but provide symptom management, planning the location of death, and the incorporation of home services or hospice.

It is a Class I indication that for patients with HF being considered for, or treated with life-extending therapies, the option for discontinuation should be anticipated and discussed through the continuum of care, including at the time of initiation, and reassessed with changing medical conditions and shifting goals of care (LOE C-LD).

Caregiver support should also be offered to family members even beyond death to help them cope with the grieving process.

A formal palliative care consult is not needed for each patient, but the primary team should exercise the above domains to improve processes of care and patient outcomes.

Specialist palliative care consultation can be useful to improve QOL and relieve suffering for patients with heart failure—particularly those with stage D HF who are being evaluated for advanced therapies, patients requiring inotropic support or temporary mechanical support, patients experiencing uncontrolled symptoms, major medical decisions, or multimorbidity, frailty, and cognitive impairment (Class 2a, LOE B). Studies have been mixed on if the palliative team itself improves quality of life and well-being so these interventions should be tailored to each patient and caregiver.

For patients with HF, execution of advanced directives can be useful to improve documentation of treatment preferences, delivery of patient-centered care, and dying in a preferred place (Class 2a, LOE C-LD).

In patients with advanced HF with expected survival < 6 months, timely referral to hospice can be useful to improve QOL (Class 2a, LOE C-LD)

Main Takeaway

In summary, the core principles of palliative care that include communication, transparency on prognosis, clarification of goals of care, shared decision-making, symptom management, and caregiver support should be integrated into each patient’s treatment plan regardless of the stage of heart failure

Guideline Loc.

Section 13, Figure 15, Table 32

Mrs. Hart is a 70-year-old woman who was admitted to the CICU two days ago for signs and symptoms consistent with cardiogenic shock. Since her admission, she has been on maximal diuretics, requiring greater doses of intravenous dobutamine. Unfortunately, her liver and renal function continue to worsen, and urine output is decreasing. A right heart catheterization reveals elevated biventricular filling pressures with a cardiac index of 1.7 L/min/m2 by the Fick method.

 

What is the next best step?

A

Continue current measures and monitor for improvement

B

Switch from dobutamine to norepinephrine

C

Place an intra-aortic balloon pump (IABP)

D

Resume guideline directed medical therapy

Explanation

The Correct answer is C – Place an intra-aortic balloon pump.

This patient is between the SCAI Shock Stages C and D with elevated venous pressures, decreased urine output, and worsening signs of hypoperfusion. She has been started on appropriate therapies, including diuresis and inotropic support. The relevant Class 2a recommendation is that in patients with cardiogenic shock, temporary MCS is reasonable when end-organ function cannot be maintained by pharmacologic means to support cardiac function (LOE B-NR). Thus, the next best step is a form of temporary MCS. IABP is appropriate to help increase coronary perfusion and offload the left ventricle. In fact, for patients who are not rapidly responding to initial shock measures, triage to centers that can provide temporary MCS may be considered to optimize management (Class 2b, LOE C-LD).

  

The guidelines further state that in patients presenting with cardiogenic shock, placement of a pulmonary arterial line may be considered to define hemodynamic subsets and appropriate management strategies (Class 2B, LOE B-NR). And so, if time allows escalation to MCS should be guided by invasively obtained hemodynamic data via PA catheterization. Several observational experiences have associated PA catheterization use with improved outcomes, particularly in conjunction with short-term MCS. Additionally, PA catheterization is useful when there is diagnostic uncertainty as to the cause of hypotension or end-organ dysfunction, particularly when the patient in shock is not responding to empiric initial measures, such as in this patient.

There are additional appropriate measures at this time that are more institution-dependent. An institutional shock team would be very helpful here as they often comprise multidisciplinary teams of heart failure and critical care specialists, interventional cardiologists, surgeons, and palliative care specialists. As such, there is a Class 2a recommendation that in patients with cardiogenic shock, management by a multidisciplinary team experienced in shock is reasonable (LOE B-NR). Most documented experiences have suggested outcomes improve after shock teams are instituted. For instance, in one such experience, using a shock team was associated with improved 30-day all-cause mortality (HR, 0.61; 95% CI, 0.41–0.93) and reduced in-hospital mortality (61.0% vs. 47.9%; P=0.041).

Choice A – Continue current measures and monitor for improvement – is incorrect. This patient has been deteriorating on current measures since admission and is at higher risk for SCAI Shock Stage E – extremis, refractory hypotension/hypoperfusion, and cardiac arrest and, therefore requires escalation of therapy

Choice B- Switch from dobutamine to norepinephrine – is incorrect. The Class 1 LOE B-NR recommendation is that in patients with cardiogenic shock, intravenous inotropic support should be used to maintain systemic perfusion and maintain end-organ performance. Dobutamine is a more potent inotropic agent than norepinephrine. Stopping dobutamine in the setting of her low cardiac index would be incorrect.

Choice D – Resume guideline-directed medical therapy – is incorrect. This patient’s shock is getting worse. The Class 1 LOE B-NR recommendation is that in patients with HFrEF, GDMT should be initiated during hospitalization after clinical stability is achieved. Restarting medications now would be premature.

Main Takeaway

In patients with cardiogenic shock, temporary MCS is reasonable when end-organ function cannot be maintained by pharmacologic means to support cardiac function.

 

Guideline Loc.

Section 9.5

Ms. V. Tea is a 55-year-old woman with a history of cardiac sarcoidosis, heart failure with mildly reduced ejection fraction (HFmrEF – EF 40%), and ventricular tachycardia with CRT-D who presents with recurrent VT. She has undergone several attempts at catheter ablation of VT in the past and previously had been trialed on amiodarone which was discontinued due to hepatotoxicity. She now continues to have episodic VT requiring anti-tachycardia pacing and ICD shocks despite medical therapy with mexiletine, metoprolol, and sotalol. Her most recent PET scan showed no active areas of inflammation. Currently, her vital signs are stable, and labs are unremarkable. What is the best next step for this patient?

A

Evaluation for heart transplant

B

Evaluation for LVAD

C

Dobutamine

D

Prednisone

E

None of the above

Explanation

The correct answer is A – evaluation for heart transplant.

For selected patients with advanced heart failure despite GDMT, cardiac transplantation is indicated to improve survival and quality of life (Class 1, LOE C-LD). Heart transplantation, in this context, provides intermediate economic value.

Clinical indicators include refractory or recurrent ventricular arrhythmias with frequent ICD shocks. Patient selection for heart transplant includes assessment of comorbidities, goals of care, and various other factors. The United Network of Organ Sharing Heart Transplant Allocation Policy was revised in 2018 with a 6-tiered system to better prioritize unstable patients and minimize waitlist mortality. VT puts the patient as a Status 2 on the transplant list. There was a contemporary analysis of patients with end-stage cardiomyopathy due to cardiac sarcoidosis, published in Journal of Cardiac Failure, in 2018 that demonstrated similar 1-year and 5-year survival after heart transplant between patients with and without cardiac sarcoidosis.

 

Choice B (evaluation for LVAD) is incorrect. While bridge to transplant with LVAD is definitely a potential next step in patients with cardiac sarcoidosis, it is not recommended in patients presenting primarily with refractory ventricular arrhythmias due to granuloma-induced scarring. In this situation, patients benefit from direct heart transplant rather than bridge to transplant LVAD approach. The same study, described before in the Journal of Cardiac Failure, also showed similar 1-year and 5-year survival after bridge-to-transplant mechanical circulatory support between patients with and without cardiac sarcoidosis. Since cardiac sarcoidosis is not just limited to the left ventricle, patients being considered for LVAD need hemodynamic assessment to determine the risk of post-LVAD RV failure.

Choice C (dobutamine) is incorrect. The patient is currently not decompensated in terms of contractility nor is showing signs of cardiogenic shock. Further, dobutamine may worsen arrhythmia burden.

Choice D (prednisone) is incorrect as there is no sign of active inflammation on the PET scan. The recurrent ventricular arrhythmias are being driven by granuloma-induced scar.

Main Takeaway

Cardiac transplantation has a Class 1 (LOE C-LD) recommendation for eligible patients with advanced HF despite GDMT to improve survival and quality of life. Specifically, direct heart transplantation is the best next step in patients with cardiac sarcoidosis and refractory ventricular arrhythmias rather than a bridge-to-transplant approach.

Guideline Loc.

Section 8.5

A 69-year-old man was referred to the cardiology clinic after being found to have a reduced left ventricular ejection fraction and left ventricular hypertrophy. For the last several months he has been experiencing progressively worsening fatigue and shortness of breath while getting to the 2nd floor in his house. He has a history of bilateral carpal tunnel syndrome and chronic low back pain. He takes no medications. On exam, his heart rate is 82 bpm, blood pressure is 86/60 mmHg, O2 saturation is 97% breathing ambient air, and BMI is 29 kg/m2. He has a regular rate and rhythm with normal S1 and S2, bibasilar pulmonary rales, and 1+ pitting edema in both legs. EKG shows normal sinus rhythm with a first-degree AV delay and low voltages. Transthoracic echocardiogram shows a moderately depressed LVEF of 35-39%, severe concentric hypertrophy with a left ventricular posterior wall thickness of 1.5 cm and strain imaging showing globally reduced longitudinal strain with apical sparring. There is also biatrial enlargement and a small pericardial effusion. A pharmacologic nuclear stress test did not reveal any perfusion defects. A gammopathy panel including SPEP, UPEP, serum and urine immunofixation studies, and serum free light chains are unrevealing. A 99mTc-Pyrophosphate scan was positive with grade 3 uptake. In addition to starting diuretics, what is the next most appropriate step for managing for this patient?

A

Start metoprolol succinate

B

Start sacubitril/valsartan

C

Perform genetic sequencing of the TTR gene

D

Perform endomyocardial biopsy

Explanation

The correct answer is C – perform genetic sequencing of the TTR gene.

 

This patient has findings which raise suspicion for cardiac amyloidosis. There are both cardiac (low voltages on EKG and echocardiogram showing marked LVH with biatrial enlargement and small pericardial effusion as well as a characteristic strain pattern) and extra-cardiac (bilateral carpal tunnel syndrome and low back pain) features to suggest amyloidosis. The diagnosis of cardiac amyloidosis requires a high index of suspicion and most commonly occurs due to a deposition of monoclonal immunoglobulin light chains (AL-CM) or transthyretin (ATTR-CM). ATTR may cause cardiac amyloidosis as either a pathogenic variant (ATTRv) or as a wild-type protein (ATTRwt).

 

Patients for whom there is a clinical suspicion for cardiac amyloidosis should have screening for serum and urine monoclonal light chains with serum and urine immunofixation electrophoresis and serum free light chains (Class 1, LOE B-NR). Immunofixation electrophoresis (IFE) is preferred because serum or urine plasma electrophoresis (SPEP or UPEP) are less sensitive. Together, measurement of serum IFE, urine IFE, and serum FLC is >99% sensitive for AL amyloidosis. Negative studies as in our patient essentially exclude AL amyloidosis from consideration.

 

In patients with high clinical suspicion for cardiac amyloidosis, without evidence of serum or urine monoclonal light chains, bone scintigraphy should be performed to confirm the presence of transthyretin cardiac amyloidosis (Class 1, LOE B-NR). As in this patient’s case, the 99mTc-Pyrophosphate scan with a grade 2/3 cardiac uptake in the absence of a serum or urinary monoclonal protein has a very high specificity and positive predictive value for ATTR-CM. This allows for a noninvasive diagnosis of ATTR-CM, obviating the need for an endomyocardial biopsy and so option D is inaccurate.

 

In patients for whom a diagnosis of transthyretin cardiac amyloidosis is made, genetic testing with TTR gene sequencing is recommended to differentiate hereditary variant from wild-type transthyretin cardiac amyloidosis (Class 1, LOE B-NR). Differentiating ATTRv from ATTRwt is important because confirmation of ATTRv would trigger genetic counseling and potential cascade screening of family members and TTR silencer therapies, such as inotersen and patisiran (currently only approved for the treatment of polyneuropathy caused by ATTRv amyloidosis).

 

Routine guideline-directed medical treatment (GDMT) for neurohormonal blockade may be poorly tolerated in patients with ATTR-CM and EF ≤40%. Due to restrictive physiology, they may be predisposed to more hypotension with ARNi, ACEi, and ARB. Similarly, patients with ATTR-CM rely on their heart rate response to preserve the cardiac output, thus BB may worsen HF symptoms. In this case, our patient already has a borderline blood pressure without these medications. Both options A and B are false.

 

Main Takeaway

In patients for whom a diagnosis of transthyretin cardiac amyloidosis is made, TTR gene sequencing is recommended to differentiate pathologic variant (ATTRv) from wild-type transthyretin cardiac amyloidosis (ATTRwt). This has implications in terms of screening for family members and management options for ATTRv.

 

For patients with ATTR-CM and EF ≤40%, GDMT may be poorly tolerated.

 

Guideline Loc.

Section 7.8, Figure 13

Mr. Gene D’aMeTi, a 53-year-old African American man with ischemic cardiomyopathy and heart failure with reduced ejection fraction (LVEF 30-35%), is recently admitted with acutely decompensated heart failure and acute kidney injury on chronic kidney disease stage III. His outpatient regiment includes sacubitril-valsartan 97-103mg BID, carvedilol 25mg BID, and hydralazine 50mg TID.

Sacubitril-valsartan was held because of worsening renal function. Despite symptomatic improvement with diuresis, his renal function continues to decline. He is otherwise well perfused & with preservation of other end organ function.

 

Throughout this hospitalization, he has steadily become more hypertensive with blood pressures persisting in the 170s/90s mmHg. What would be an appropriate adjustment to his medication regimen at this time?

A

Resume Losartan only

B

Start Amlodipine

C

Increase current Hydralazine dose

D

Start Isosorbide dinitrate therapy

E

Both C & D

Explanation

The correct answer is E – both increasing the current hydralazine dose (C) and starting isosorbide dinitrate therapy (D).

 

Although ACEI/ARB therapy (choice A) has shown a mortality and morbidity benefit in HFrEF, caution should be used in patients with renal insufficiency. In this patient with ongoing decline in renal function, RAAS-inhibiting therapies (ACEi, ARB, ARNI, MRA) should be avoided. In this case, as his RAAS-I has been stopped, it would be reasonable to increase current therapies to target doses (or nearest dose tolerated), as these demonstrated both safety and efficacy in trials (Class 1, LOE A). Considering that his high dose ARNI was stopped, it is unlikely that either hydralazine or isosorbide dinitrate alone, even at maximal doses, would be sufficient to control his blood pressure (Options C and D, respectively). Interestingly, in the original study by Massie et. Al (1977), the decision was made to combine these therapies as the result was thought to be superior to either medication alone. ISDN would provide preload reduction, while Hydralazine would decrease afterload. Consequently, we do not have data looking at the individual benefit of either medication in isolation.

 

In self-identified African Americans with NYHA class III or IV HFrEF already on optimal GDMT, the addition of hydralazine & isosorbide dinitrate is recommended to improve symptoms and reduce mortality and morbidity (Class 1, LOE A). In this case, as the patient has evidence of progressive renal disfunction, we are limited in using traditional RAAS-I, such as ACEI, ARB, or ARNI. In patients with current or previously symptomatic HFrEF who cannot be given first-line agents (like ARNi, ACEi, ARB) due to intolerance or renal insufficiency, combination therapy of hydralazine & isosorbide dinitrate might be considered to reduce morbidity and mortality (Class 2b, LOE C-LD).

 

Dihydropyridine calcium channel blockers such as Amlodipine (choice B) are not recommended for treatment of HFrEF (COR 3, LOE A), though may be considered for treating elevated blood pressure despite optimization of GDMT.

Main Takeaway

In self-identified African Americans, the addition of hydralazine & isosorbide dinitrate to GDMT has additional mortality & morbidity benefits.

 

Should a patient have drug intolerances or renal dysfunction that precludes the use of ACEi/ARB/ARNi, hydralazine & isosorbide dinitrate is a reasonable alternative.

Guideline Loc.

·       Section 7.3.5-8

·       Table 14, Table 15

Which of the following sentences regarding diuretics in the management of heart failure is correct?

A

In HF patients with minimal congestive symptoms, medical management with diuretics alone is sufficient to improve outcomes.

B

Prescribing a loop diuretic on discharge after a HF hospitalization may improve short term mortality and HF rehospitalization rates.

C

The combination of thiazide (or thiazide-like) diuretics with loop diuretics is preferred to higher doses of loop diuretics in patients with HF and congestive symptoms.

D

The maximum daily dose of furosemide is 300 mg.

Explanation

Choice B in correct. The guidelines give a Class 1 recommendation for diuretics in HF patients who have fluid retention to relieve congestion, improve symptoms, and prevent worsening heart failure. Recent data from the non-randomized OPTIMIZE-HF (Organized Program to Initiate Lifesaving Treatment in Hospitalized Patients with Heart Failure) registry revealed reduced 30-day all-cause mortality and hospitalizations for HF with diuretic use compared with no diuretic use after hospital discharge for HF.

Choice A is incorrect. With the exception of mineralocorticoid receptor antagonists (MRAs), the effects of diuretics on morbidity and mortality are uncertain. As such, diuretics should not be used in isolation, but always combined with other GDMT for HF that reduce hospitalizations and prolong survival.

Choice C is incorrect. The use of a thiazide or thiazide-like diuretic (e.g., metolazone) in combination with a loop diuretic inhibits compensatory distal tubular sodium reabsorption, leading to enhanced natriuresis. In a propensity-score matched analysis in patients with hospitalized HF, the addition of metolazone to loop diuretics was found to increase the risk for hypokalemia, hyponatremia, worsening renal function, and mortality, whereas use of higher doses of loop diuretics was not found to adversely affect survival. The guidelines recommend that the addition of a thiazide (e.g., metolazone) to treatment with a loop diuretic should be reserved for patients who do not respond to moderate- or high-dose loop diuretics to minimize electrolyte abnormalities (Class I, LOE B-NR).

 

Choice D is incorrect. The guidelines recommend a maximum total daily dose of 600mg of furosemide or 10mg of bumetanide or 200mg of torsemide.

Main Takeaway

In summary, diuretics are recommended in heart failure patients who have fluid retention to relieve congestion, improve symptoms, and prevent worsening heart failure. Maintenance diuretics on HF hospitalization discharge may help prevent recurrent HF hospitalizations. They should be used in combination with other GDMT to improve HF outcomes. Combining loop and thiazide diuretics may cause serious electrolyte abnormalities and should be reserved for patients who do not respond to moderate- or high-dose loop diuretics.

 

Guideline Loc.

Section 7.2, Table 12

 

A 45-year-old man presents to cardiology clinic to establish care. He has had several months of progressive dyspnea on exertion while playing basketball. He also reports intermittent palpitations for the last month. Two weeks ago, he passed out while playing and attributed this to exertion and dehydration. He denies smoking and alcohol intake.

 

Family history is significant for sudden cardiac death in his father at the age of 50 years. Autopsy has shown a thick heart, but he is unaware of the exact diagnosis. He has two children, ages 12 and 15 years old, who are healthy.

 

Vitals signs are blood pressure of 124/84 mmHg, heart rate of 70 bpm, and normal respiratory rate. On auscultation, a systolic murmur is present at the left lower sternal border. A 12-lead ECG showed normal sinus rhythm with signs of LVH and associated repolarization abnormalities. Echocardiography reveals normal LV chamber volume, preserved LVEF, asymmetric septal hypertrophy with wall thickness up to 16mm, systolic anterior motion of the anterior mitral valve leaflet with 2+ eccentric posteriorly directed MR, and resting LVOT gradient of 30mmHg which increases to 60mmHg on Valsalva.

 

You discuss your concern for an inherited cardiomyopathy, namely hypertrophic cardiomyopathy. In addition to medical management of his symptoms and referral to electrophysiology for ICD evaluation, which of the following is appropriate at this time?

Order blood work for genetic testing

Referral for genetic counseling

Cardiac MRI

Coronary angiogram

All of the above

Explanation  

The correct answer is B – referral for genetic counseling. 

Several factors on clinical evaluation may indicate a possible underlying genetic cardiomyopathy. Clues may be found in:

·       Cardiac morphology – marked LV hypertrophy, LV noncompaction, RV thinning or fatty replacement on imaging or biopsy

·       12-lead ECG – abnormal high or low voltage or conduction, and repolarization, altered RV forces

·       Presence of arrhythmias – frequent NSVT or very frequent PVCs, sustained VT or VF, early onset AF, early onset conduction disease

·       Extracardiac features – skeletal myopathy, neuropathy, cutaneous stigmata, and other possible manifestations of specific syndromes

In select patients with nonischemic cardiomyopathy, referral for genetic counseling and testing is reasonable to identify conditions that could guide treatment for patients and family members (Class 2a, LOE B-NR). In first-degree relatives of selected patients with genetic or inherited cardiomyopathies, genetic screening and counseling are recommended to detect cardiac disease and prompt consideration of treatments to decrease HF progression and sudden death (Class 1, LOE B-NR).

No controlled studies have shown clinical benefits of genetic testing for cardiomyopathy, but genetic testing contributes to risk stratification and has implications for treatment, currently most often for decisions regarding defibrillators for primary prevention of sudden death and regarding exercise limitation for hypertrophic cardiomyopathy and the desmosomal variants. Consultation with a trained counselor before and after genetic testing helps patients to understand and weigh the implications of possible results for their

own lives and those of family members, including possible discrimination on the basis of genetic information. Unless shown to be free of the genetic variant(s) implicated in the proband, first-degree relatives of affected probands should undergo periodic screening with echocardiography and electrocardiography.

In this patient with likely hypertrophic cardiomyopathy, a family history of sudden cardiac death, recent unexplained syncope, and two children, a referral for genetic counseling is appropriate at this time. However, option A is incorrect because ordering genetic testing without meeting a trained counselor is not advised. 

Main Takeaway 

Patients with the possibility of genetic cardiomyopathies should be referred to trained genetic counselors before and after genetic testing to understand the implications of the testing and results.

Guideline Loc. 

Section 4.3, Table 7

A 50-year-old man with a history of type 2 diabetes mellitus, persistent atrial fibrillation, coronary artery disease with prior remote percutaneous coronary intervention, and ischemic cardiomyopathy with HFrEF (LVEF 38%) presents to your outpatient clinic. He denies dyspnea on exertion, orthopnea, bendopnea, paroxysmal nocturnal dyspnea, or peripheral edema. His heart rate is irregularly irregular at 112 beats per minute and blood pressure is 112/67 mmHg. Routine laboratory studies reveal a hemoglobin A1c of 7.7%. Which of the following medications should not be used to control this patient’s comorbidities?

A

Metoprolol succinate

B

Verapamil

C

Dapagliflozin

D

Pioglitizone

E

Both B and D

Explanation

The correct answer is E – both verapamil and pioglitazone should be avoided here.

Both verapamil and pioglitizone are associated with harm in patients with LVEF < 50% (Class 3: Harm). Verapamil and diltiazem are non-dihydropyridine calcium channel blockers. These medications can cause negative inotropic effects through inhibition of calcium influx and may be harmful in this patient population. Pioglitizone belongs to a class of diabetic medications known as the thiazolidinediones. This class of medications may increase the risk of fluid retention, heart failure, and hospitalization in patients with LVEF of less than 50%.

Metoprolol succinate, and other evidence-based beta blockers, have a Class 1 recommendation for patients with reduced ejection fraction ≤ 40% to prevent symptomatic heart failure and reduce mortality. It may additionally help with rate control in this patient with atrial fibrillation and rapid ventricular response.

SGLT2 inhibitors including dapagliflozin have a Class I recommendation for patients with symptomatic chronic HFrEF to reduce hospitalization for HF and cardiovascular mortality, irrespective of the presence of type 2 diabetes (Class 1, LOE A). They also have a Class I recommendation in patients with type 2 diabetes and either established CVD or at high cardiovascular risk to prevent hospitalization for HF (Class 1, LOE A). Our patient has asymptomatic, or pre-HF (Stage B) heart failure with poorly controlled diabetes, and so use of an SGLT2 inhibitor would be appropriate.

Main Takeaway

Non-dihydropyridine calcium channel blockers and thiozolidinediones

both have Class 3 recommendations for harm in patients with reduced LV systolic dysfunction.

Guideline Loc.

Section 6.1 and 7.3

 

Mr. E. Regular is a 61-year-old man with a history of HFrEF due to non-ischemic cardiomyopathy (latest LVEF 40% after >3 months of optimized GDMT) and persistent atrial fibrillation. He has no other medical history. He has been on metoprolol and apixaban and has also undergone multiple electrical cardioversions and catheter ablations for atrial fibrillation but remains symptomatic with poorly controlled rates. His blood pressure is 105/65 mm Hg. HbA1c is 5.4%. Which of the following is a reasonable next step in the management of his atrial fibrillation?

A

Anti-arrhythmic drug therapy with amiodarone. Stop apixaban.

B

Repeat catheter ablation for atrial fibrillation. Stop apixaban.

C

AV nodal ablation and RV pacing. Shared decision-making regarding anticoagulation.

D

AV nodal ablation and CRT device. Shared decision-making regarding anticoagulation.

Explanation

The correct answer is D – AV nodal ablation and CRT device along with shared decision-making regarding anticoagulation.”

Maintaining sinus rhythm and atrial-ventricular synchrony is helpful in patients with heart failure given the hemodynamic benefits of atrial systole for diastolic filling and having a regularized rhythm.

Recent randomized controlled trials suggest that catheter-based rhythm control strategies are superior to rate control and chemical rhythm control strategies with regards to outcomes in atrial fibrillation. For patients with heart failure and symptoms caused by atrial fibrillation, ablation is reasonable to improve symptoms and quality of life (Class 2a, LOE B-R). However, Mr. Regular has already had multiple failed attempts at ablations (option B).

For patients with AF and LVEF ≤50%, if a rhythm control strategy fails or is not desired,

and ventricular rates remain rapid despite medical therapy, atrioventricular nodal ablation with implantation of a CRT device is reasonable (Class 2a, LOE B-R). The PAVE and BLOCK-HF trials suggested improved outcomes with CRT devices in these patients.

RV pacing following AV nodal ablation has also been shown to improve outcomes in patients with atrial fibrillation refractory to other rhythm control strategies. In patients with EF >50%, there is no evidence to suggest that CRT is more beneficial compared to RV-only pacing. However, RV pacing may produce ventricular dyssynchrony and when compared to CRT in those with reduced EF (≤ 50%), CRT produced more benefit (Option C).

Although adjustments in antiarrhythmic medications and repeat ablation may be considered, these are unlikely to provide long-term benefit to Mr. E. Regular, who already failed antiarrhythmic regimens and multiple attempts at cardioversion and ablation (Options A, B).

In patients with chronic heart failure and atrial fibrillation, the decision to use anticoagulation for the prevention of cerebrovascular events is generally based on the CHA2DS2-VASc score. Mr. Regular’s CHA2DS2-VASc score is 1 (+1 for HF, no points for: hypertension, age 65-74 or ≥75, diabetes, stroke/TIA/TE, vascular disease, female gender). Chronic anticoagulation therapy is recommended for patients with CHA2DS2-VASc scores ≥2 for men and ≥3 for women (Class 1, LOE A). Therefore, based on the CHA2DS2-VASc score alone, Mr. Regular would not necessarily warrant anticoagulation. However, HF is a hypercoagulable state and serves as an independent risk factor for stroke, systemic embolism, and mortality in the setting of AF. In patients

with HF and a CHA2DS2-VASc score of 1, those with AF had a 3-fold higher risk compared with individuals without concomitant AF. Because HF is a risk factor, additional risk factors may not be required to support the use of anticoagulation in patients with HF, and the decision to anticoagulate can be individualized according to risk versus benefit. The guidelines give a Class 2a recommendation for chronic anticoagulation in men and women with chronic HF and permanent-persistent-paroxysmal AF who have no additional risk factors (LOE B-NR). Therefore, decisions regarding anticoagulation in this context should incorporate patient values, comorbidities, and informed shared decision making.  

Main Takeaway

In summary, the “ablate and pace” strategy of AV nodal ablation and CRT device implantation improve outcomes in patients with heart failure with reduced LVEF and atrial fibrillation refractory to chemical and catheter-based rhythm control strategies and failure of rate control options.

Guideline Loc.

Section 10.2

Mrs. Hart is a 63-year-old woman with a history of non-ischemic cardiomyopathy and heart failure with reduced ejection fraction (LVEF 20-25%) presenting with 5 days of worsening dyspnea and orthopnea.

 

At home, she takes carvedilol 12.5mg BID, sacubitril-valsartan 24-46mg BID, empagliflozin 10mg daily, and furosemide 40mg daily.

 

On admission, her exam revealed a blood pressure of 111/79 mmHg, HR 80 bpm, and SpO2 94%. Her cardiovascular exam was significant for a regular rate and rhythm with an audible S3, JVD to 13 cm H2O, bilateral lower extremity pitting edema with warm extremities and 2+ pulses throughout.  What initial dose of diuretics would you give her?

A

Continue home Furosemide 40 mg PO

B

Start Metolazone 5 mg PO

C

Start Lasix 100 mg IV

D

Start Spironolactone

Explanation

The correct answer is C – start Furosemide 100 mg IV.

This is the most appropriate choice because patients with HF admitted with evidence of significant fluid overload should be promptly treated with intravenous loop diuretics to improve symptoms and reduce morbidity (Class 1, LOE B-NR). Intravenous loop diuretic therapy provides the most rapid and effective treatment for signs and symptoms of congestion. Titration of diuretics has been described in multiple recent trials of patients hospitalized with HF,

often initiated with at least 2 times the daily home diuretic dose (mg to mg) administered intravenously. Titration to achieve effective diuresis may require doubling of initial doses, adding a thiazide diuretic, or adding an MRA that has diuretic effects in addition to its cardiovascular benefits.

Choice A is incorrect as continuing oral loop diuretics is not recommended for acute decongestion. Moreover, Ms. Hart has become congested despite her home, oral diuretic regimen.

Choice B and D are incorrect as starting a thiazide diuretic or a mineralocorticoid receptor antagonist are not first-line therapy for acute HF. Rather, in patients hospitalized with HF when diuresis is inadequate to relieve symptoms and signs of congestion, it is reasonable to intensify the diuretic regimen using either: a. higher doses of intravenous loop diuretics; or b. addition of a second diuretic (Class 2a, LOE B-NR).

After instituting intravenous loop diuretic therapy, escalating attempts to achieve net diuresis include serial doubling of intravenous loop diuretic doses, which can be done by bolus or infusion, and sequential nephron blockade with addition of a thiazide diuretic, as detailed specifically in the protocol for the diuretic arms of the CARRESS and ROSE trials.

MRAs have mild diuretics properties and the addition of MRAs can help with diuresis in addition to significant cardiovascular benefits in patients with HF.

For patients hospitalized with HF, therapy with diuretics and other guideline-directed medications should be titrated with a goal to resolve clinical evidence of congestion to reduce symptoms and rehospitalizations (Class 1, LOE B-NR).

For patients requiring diuretic treatment during hospitalization for HF, the discharge regimen should include a plan for adjustment of diuretics to decrease rehospitalizations (Class 1, LOE B-NR).

Main Takeaway

Patients admitted with acute HF should be promptly treated with intravenous loop diuretics. If current level of diuresis becomes inadequate to relieve symptoms and signs of congestion, it is reasonable to intensify the diuretic regimen using either: higher doses of intravenous loop diuretics or addition of a second diuretic (e.g., thiazide or MRA). All patients should have their diuretic regimen updated on discharge.

Guideline Loc.

Section 9.3

You are taking care of a 34-year-old man with chronic systolic heart failure from NICM with LVEF 20% s/p CRT-D. The patient was admitted 1 week prior with acute decompensated heart failure. Despite intravenous diuretics the patient developed acute kidney injury, and ultimately placed on intravenous inotropes on which he now seems dependent. He has been following up with an advanced heart failure specialist as an outpatient and has been undergoing evaluation for heart transplantation, which was subsequently completed in the hospital.

 

His exam is notable for an elevated JVP, a III/VI holosystolic murmur, and warm extremities with bilateral 1+ edema. His most recent TTE shows LVEF 20%, moderate MR, moderate-severe TR and estimated RVSP 34 mmHg. His most recent laboratory data shows Na 131 mmol/L, Cr 1.2 mg/dL, and lactate 1.6 mmol/L. Pulmonary artery catheter shows RA 7 mmHg, PA 36/15 mmHg, PCWP 12 mmHg, CI 2.4 L/min/m2 and SVR 1150 dynes*sec/cm5.

 

The patient was presented at transplant selection committee and approved for listing for orthotopic heart transplant. What is the most appropriate next step in the management of this patient?

A

Refer patient for transcatheter edge-to-edge repair for MR

B

Continue IV inotropes as a bridge-to-transplant

C

Refer patient for tricuspid valve replacement

D

Initiate 1.5L fluid restriction

Explanation

The correct answer is B – continue IV inotropes as a bridge-to-transplant.

Positive inotropic agents may improve hemodynamic status, but have not been shown to improve survival in patients with HF. These agents may help HF patients who are refractory to other therapies and are suffering consequences from end-organ-hypoperfusion.

Our patient is admitted with worsening advanced heart failure requiring intravenous inotropic support. He has been appropriately evaluated and approved for heart transplant. He has demonstrated the requirement of continuous inotropic support to maintain perfusion. In patients such as this with advanced (stage D) HF refractory to GDMT and device therapy who are eligible for and awaiting MCS or cardiac transplantation, continuous intravenous inotropic support is reasonable as “bridge therapy” (Class 2a, LOE B-NR).

Continuous IV inotropes also have a Class 2b indication (LOE B-NR) in select patients with stage D HF despite optimal GDMT and device therapy who are ineligible for either MCS or cardiac transplantation, as palliative therapy for symptom control and improvement in functional status.

Conversely, long-term use of either continuous or intermittent intravenous inotropic agents, for reasons other than palliative care or as a bridge to advanced therapies, is potentially harmful (Class 3: Harm, LOE B-R).

As of yet there is lack of clear evidence suggesting the benefit of one inotrope over another. To minimize adverse effects, the lowest possible dose of inotropes should be used, although the potential for development of tachyphylaxis should be acknowledged and the choice/dose of agent may need to be changed over time for longer periods of use. In addition, the ongoing need for inotropic support and the possibility of discontinuation should be regularly assessed.

Although guidelines give a Class 2a recommendation for transcatheter edge-to-edge MV repair in patients with reduced EF and severe MR with persistent symptoms despite GDMT, this patient’s MR was graded as moderate on his most recent TTE and as such, he would not be an appropriate candidate for TEER.

Although guidelines give a Class 1 recommendation for multidisciplinary management of patients with HF and VHD, as well as referral for consideration of intervention in patients with refractory TR, there are currently no guideline recommendations supporting surgical TVR in advanced HF patients with TR.

Although fluid restriction has been associated with modest improvements in hyponatremia in patients with advanced HF, the clinical benefits of this therapy remain uncertain and as such was given a Class 2b recommendation in the clinical guidelines.

 

Main Takeaway

Continuous intravenous inotropic support can be considered in patients with advanced heart failure refractory to GDMT who are awaiting durable MCS or heart transplant as “bridge therapy” (Class 2a) or for palliative therapy in patients with advanced HF who are ineligible for MCS/transplant (Class 2b), but is potentially harmful for long-term use for reasons beyond palliation or bridge to advanced therapies (class 3 recommendation).

Guideline Loc.

Section 8.3

Table 20

Ms. Smith is a 56-year-old woman following up in the cardiology clinic for a history of heart failure with reduced ejection fraction. Two years ago, she was diagnosed with non-ischemic cardiomyopathy with a left ventricular ejection fraction (LVEF) of 30%. Over time, she was initiated and optimized on guideline directed medical therapy. She is currently on Carvedilol 12.5 mg BID, Sacubitril/Valsartan 49/51 mg BID, Spironolactone 25 mg daily, Empagliflozin 10 mg daily, and Furosemide PRN for weight gain.

 

On today’s visit, her BP is 110/80 mmHg, and her HR is 67 bpm. Labs show a creatinine of 0.9 mg/dL, potassium of 5.1 mEq/L, NT-proBNP of 150 ng/L, and a HbA1c of 5.8%. Follow up transthoracic echocardiogram showed an improvement in LVEF to 55%. What are the most appropriate therapy recommendations for Ms. Smith?

A

Discontinue spironolactone

B

Discontinue empagliflozin

C

Decrease the dose of carvedilol

D

Continue current therapy

The correct answer is D – continue current therapy.

The patient described above was initially diagnosed with HFrEF and experienced significant symptomatic improvement with GDMT, so she now has heart failure with improved ejection fraction (HFimpEF). In patients with HFimpEF after treatment, GDMT should be continued to prevent relapse of HF and LV dysfunction, even in patients who may become asymptomatic (Class 1, LOE B-R). Although symptoms, functional capacity, LVEF and reverse remodeling can improve with GDMT, structural abnormalities of the LV and its function do not fully normalize, causing symptoms and biomarker changes to persist or recur if treatment is deescalated. Improvements in EF do not always reflect sustained recovery; rather, they signify remission.

 

Of note, HF relapse can be defined by at least 1 of the following:

o   A drop in the EF by >10% and to < 50%

o   An increase in LVEDV by >10% and to higher than the normal range

o   A 2-fold rise in NT-proBNP concentration and to > 400 ng/L

o   Clinical evidence of HF on examination

Choice A is incorrect as it would be incorrect to discontinue spironolactone. A potassium of 5.1 is still within the acceptable limit in a patient who has been on Spironolactone for two years, and this medication is an important part of GDMT for HFrEF.

 

Despite the improvement in Hb A1c, empagliflozin should be continued for heart failure with improved ejection fraction, as it is part of routine GDMT of HFrEF even in the absence of diabetes. Choice B is thus incorrect.


Similarly, carvedilol should be continued at the same dose as the patient’s heart rate is within the desired range. Furthermore, all GDMT should be continued in patients with HFimpEF, as emphasized above. Choice C is therefore also incorrect.

In patients with HFimpEF after treatment, GDMT should be continued to prevent relapse of HF and LV dysfunction, even in patients who may become asymptomatic. (Class 1, LOE B-R).

Section 7.6.2

Ms. Betty Blocker is a 60-year-old woman with a history of alcohol-related dilated cardiomyopathy who presents for follow up. She has been working hard to improve her health and is glad to report that she has just reached her 5-year sobriety milestone. Her current medications include metoprolol succinate 100mg daily, sacubitril-valsartan 97-103mg BID, spironolactone 25mg daily, and empagliflozin 10mg daily. She is asymptomatic at rest and up to moderate exercise, including chasing her grandchildren around the yard. A recent transthoracic echocardiogram shows recovered LVEF from previously 35% now to 60%. Ms. Blocker does not love taking so many medications and asks about discontinuing her metoprolol. Which of the following is the most appropriate response to Ms. Blocker’s request?

A

Since the patient is asymptomatic, metoprolol can be stopped without risk

B

Stopping metoprolol increases this patient’s risk of worsening cardiomyopathy regardless of current LVEF or symptoms

C

Because the LVEF is now >50%, the patient is now classified as having HFpEF and beta-blockade is no longer indicated; metoprolol can be safely discontinued

D

Metoprolol should be continued, but it is safe to discontinue either ARNi or spironolactone

Explanation

The correct answer is B – stopping metoprolol would increase her risk of worsening cardiomyopathy.

Heart failure tends to be a chronically sympathetic state. The use of beta-blockers (specifically bisoprolol, metoprolol succinate, and carvedilol) targets this excess adrenergic output and has been shown to reduce the risk of death in patients with HFrEF. Beyond their mortality benefit, beta-blockers can improve LVEF, lessen the symptoms of HF, and improve clinical status. Therefore, in patients with HFrEF, with current or previous symptoms, use of 1 of the 3 beta blockers proven to reduce mortality (e.g., bisoprolol, carvedilol, sustained-release metoprolol succinate) is recommended to reduce mortality and hospitalizations (Class 1, LOE A). Beta-blockers in this setting provide a high economic value.

Table 14 of the guidelines provides recommendations for target doses for GDMT medications. Specifically for beta blockers, those targets are 25-50mg twice daily for carvedilol (or 80mg once daily for the continuous release formulation), 200mg once daily for metoprolol succinate, and 10mg once daily for bisoprolol.

While we should be cognizant of pill-burden and other barriers to our patients’ quality of life, we must counsel them about the risks of discontinuing any element of guideline directed medical therapy (GDMT). The 2022 heart failure guidelines recommend the long-term use of beta blockers for patients diagnosed with HFrEF, even if symptoms improve (Option A). Conversely, long-term treatment should also be maintained even if symptoms do not improve to reduce the risk of major cardiovascular events. Importantly, the abrupt withdrawal of beta blockers can lead to clinical deterioration.

Our patient here has heart failure with improved ejection fraction (HFimpEF) defined as having a previous LVEF ≤ 40% and a ≥ 10-point increase from baseline with a follow-up measurement of LVEF > 40%. HFimpEF is distinct from HFpEF and was proposed in the “Universal Definition and Classification of Heart Failure” by Bozkurt et al published in JCF 2021 in order to distinguish those who benefit from continued GDMT. Accordingly, in patients with HFimpEF after treatment, GDMT should be continued to prevent relapse of HF and LV dysfunction, even in patients who may become asymptomatic (Class 1, LOE B-R). While GDMT may improve symptoms, functional capacity, LVEF, and reverse remodeling in patients with HFrEF, these favorable changes do not reflect full and sustained recovery but rather remission with susceptibility to worsening with GDMT withdrawal. Therefore, stopping any element of her GDMT (BB, ARNi, or MRA) would be incorrect (Options A, C, D).

Main Takeaway

In patients with HFrEF who experience improvement in heart failure symptoms and cardiac function on GDMT (develop HFimpEF), it is important to continue optimizing GDMT to prevent relapse, even if asymptomatic.

Guideline Loc.

Section 7.3.2

Section 7.3.8, Table 14
Section 7.6.2

Ms. M is a 36-year-old G1P1 woman 6 months postpartum who was diagnosed with peripartum cardiomyopathy at the end of her pregnancy. She is presenting for a follow up visit today and notes that while her leg edema has resolved, she continues to have dyspnea when carrying her child up the stairs. She also describes significant difficulty sleeping, though denies orthopnea, and notes she is not participating in hobbies she had previously enjoyed. She is currently prescribed a regimen of sacubitril-valsartan, metoprolol succinate, spironolactone, and empagliflozin.  What are the next best steps?

A

Screen for depression

B

Counsel her to follow a strict low sodium diet with goal of < 1.5g Na daily

C

Recommend exercise therapy and refer to cardiac rehabilitation

D

A & C

Explanation

 

The correct answer is D – both A (screening for depression) and C (referring to cardiac rehabilitation) are appropriate at this time.

Choice A is correct. Depression is a risk factor for poor self-care, rehospitalization, and all-cause mortality among patients with HF. Interventions that focus on improving HF self-care have been reported

to be effective among patients with moderate/severe depression with reductions in hospitalization and mortality risk. Social isolation, frailty, and marginal health literacy have similarly been associated with poor HF self-care and worse outcomes in patients with HF. Therefore, in adults with HF, screening for depression, social isolation, frailty, and low health

literacy as risk factors for poor self-care is reasonable to improve management (Class 2a, LOE B-NR). 

Choice C is correct. In patients with HF, cardiac rehabilitation has a Class 2a recommendation (LOE B-NR) to improve functional capacity, exercise tolerance, and health-related QOL; exercise training (or regular physical activity) for those able to participate has a Class 1 recommendation (LOE A) to improve functional status, exercise performance, and QOL.

Choice B is incorrect. For patients with stage C HF, avoiding excessive sodium intake is reasonable to reduce congestive symptoms (Class 2a, LOE C-LD). However, strict sodium restriction does not have strong supportive data and is not recommended. There are ongoing studies to better understand the impact of sodium restriction on clinical outcomes and quality of life. The AHA currently recommends a reduction of sodium intake to <2300 mg/d for general cardiovascular health promotion; however, there are no trials to support this level of restriction in patients with HF.

Main Takeaway

Depression is a risk factor for poor HF self-care and worse outcomes in patients with heart failure and so it is reasonable to screen for depression in these patients. Exercise therapy and cardiac rehabilitation have been shown to improve outcomes in HF patients. While avoiding excess sodium intake is reasonable in HF patients to reduce congestive symptoms, there is no specific strict sodium level recommended.

Guideline Loc.

Section 7.1

Ms. AH is a 48-year-old woman who presents with a 3-month history of progressively worsening exertional dyspnea and symmetric bilateral lower extremity edema. She has no history of recent upper respiratory symptoms or chest pain.

She denies any tobacco, alcohol, or recreational drug use. There is no family history of premature CAD or HF.

On exam, her blood pressure is 110/66 mmHg, heart rate is 112 bpm, and respiration rate is 18 breaths/min with oxygen saturation of 98% on ambient room air. She has jugular venous distention of about 12cm H2O, bibasilar crackles, an S3 heart sound, and bilateral lower extremity edema.

Complete blood count, serum electrolytes, kidney function tests, liver chemistry tests, glucose level, iron studies, and lipid levels are unremarkable.

An electrocardiogram shows sinus tachycardia with normal intervals and no conduction delays. A transthoracic echocardiogram demonstrates a left ventricular ejection fraction of 25%, normal right ventricular size and function, and no valvular abnormalities.

Which of the following diagnostic tests has a Class I indication for further evaluation?

A

Cardiac catheterization

B

Referral for genetic counseling

C

Thyroid function studies

D

Cardiac MRI

Explanation 

The correct answer is C – thyroid function studies have a Class 1 indication for the evaluation of HF. 

 

The common causes of HF include coronary artery disease, hypertension, and valvular heart disease. Other causes may include arrhythmia-associated, toxic, inflammatory, metabolic including both endocrinopathies and nutritional, infiltrative, genetic, stress induced, peripartum, and more. It is important to evaluate for the etiology of a given patient’s heart failure as diagnosis may have implications for treatment, counseling, and family members.

 

For patients who are diagnosed with HF, laboratory evaluation should include complete blood count, urinalysis, serum electrolytes, blood urea nitrogen, serum creatinine, glucose, lipid profile, liver function tests, iron studies, and thyroid-stimulating hormone to optimize management (Class 1, LOR C-EO). These studies provide important information regarding comorbidities, suitability for and adverse effects of treatments, potential causes or confounders of HF, and severity and prognosis of HF.

HF is often caused by coronary atherosclerosis, and evaluation for ischemic heart disease can help in determining the presence of significant coronary artery disease (CAD). Noninvasive stress imaging with echocardiography or nuclear scintigraphy can be helpful in identifying patients likely to have obstructive CAD. Invasive or computed tomography coronary angiography can detect and characterize the extent of CAD. Therefore, in patients with HF, an evaluation for possible ischemic heart disease can be useful to identify the cause and guide management (Class 2a, LOE B-NR).

Familial cardiomyopathy is increasingly recognized and may be the underlying etiology of patients previously classified as having idiopathic dilated cardiomyopathy. A detailed family history may provide the first clue to a genetic basis. A 3-generation family pedigree obtained by genetic health care professionals improved the rate of detection of a familial process as compared with routine care. Furthermore, a family history of cardiomyopathy, as determined by a 3-generation pedigree analysis, was associated with findings of gadolinium enhancement on cardiac magnetic resonance imaging (MRI) and increased major adverse cardiac events. The possibility of an inherited cardiomyopathy provides the impetus for cascade screening of undiagnosed family members, thereby potentially avoiding preventable adverse events in affected relatives by implementation of GDMT and other management that otherwise would not be initiated. Therefore, in patients with cardiomyopathy, a 3-generation family history should be obtained or updated when assessing the cause of the cardiomyopathy to identify possible inherited disease (Class 1, LOE B-NR). In selecting patients with nonischemic cardiomyopathy, referral for genetic counseling and testing is reasonable to identify conditions that could guide treatment for patients and family members (Class 2a, LOE B-NR).

CMR provides noninvasive characterization of the myocardium that may provide insights into HF cause. Registry data show that CMR findings commonly impact patient care management and provide diagnostic information in patients with suspected myocarditis or cardiomyopathy. However, routine screening with CMR is not recommended. The OUTSMART HF trial recently demonstrated routine cardiac MRI use did not yield more specific HF causes than a selective strategy based on echocardiographic and clinical findings. The guidelines give a Class 2a recommendation for the use of CMR in diagnosis or management in patients with HF or cardiomyopathy (LOE B-NR).  

Main Takeaway

The common causes of HF include ischemic heart disease, hypertension, and valvular heart disease. When a patient presents with new-onset heart failure, a complete initial evaluation including laboratory testing for potentially reversible causes such as thyroid disease, or other endocrine, metabolic, and nutritional causes should be performed.

Guideline Loc.

Section 3.2, 4.1, 4.3, and 4.4

Table 5

A 63-year-old man with CAD s/p CABG 3 years prior, type 2 diabetes mellitus, hypertension, obesity, and tobacco use disorder presents for routine follow-up. His heart rate is 65 bpm and blood pressure is 125/70 mmHg. On physical exam, he is breathing comfortably with clear lungs, with normal jugular venous pulsations, a regular rate and rhythm without murmurs or gallops, and no peripheral edema. Medications include aspirin 81mg daily, atorvastatin 80mg daily, lisinopril 20mg daily, and metformin 1000mg BID. His latest hemoglobin A1C is 7.5% and recent NT-proBNP was normal. His latest transthoracic echocardiogram showed normal biventricular size and function. Which of the following would be a good addition to optimize his medical therapy?

 

A

DPP-4 inhibitor

B

Dihydropyridine calcium channel blocker

C

SGLT2 inhibitor

D

Furosemide

Explanation

The correct answer is C: SGLT2 inhibitor.

This patient is at risk for HF (Stage A) given absence of signs or symptoms of heart failure but presence of coronary artery disease and several risk factors including diabetes, hypertension, obesity, and tobacco smoking. At this stage, the focus should be on risk factor modification and prevention of disease onset.

Healthy lifestyle habits such as maintaining regular physical activity; normal weight, blood pressure, and blood glucose levels; healthy dietary patterns, and not smoking have been associated with a lower lifetime risk of developing HF.

Multiple RCTs in patients with type 2 diabetes who have established CVD or are at high risk for CVD, have shown that SGLT2i prevent HF hospitalizations compared with placebo. The benefit for reducing HF hospitalizations in these trials predominantly reflects primary prevention of symptomatic HF, because only approximately 10% to 14% of participants in these trials had HF at baseline.

As such, in patients with type 2 diabetes and either established CVD or at high cardiovascular risk, SGLT2i should be used to prevent hospitalizations for HF (Class 1, LOE A).

The mechanisms for the improvement in HF events from SGLT2i have not been clearly elucidated but seem to be independent of glucose lowering. Proposed mechanisms include reductions in plasma volume, cardiac preload and afterload, alterations in cardiac metabolism, reduced arterial stiffness, and interaction with the Na+/H+ exchanger. SGLT2i are generally well tolerated, but these agents have not been evaluated in those with severe renal impairment (estimated glomerular filtration rate [eGFR] <25 mL/min/1.73 m2).

Main Takeaway

It is important to identify patients who are at risk for HF (Stage A) and focus on risk factor optimization to prevent disease onset and progression.

Guideline Loc.

Section 5.1

Ms. Augustin is a 33 y/o G1P1 woman from Haiti who seeks counseling regarding family planning as she and her husband dream of a second child.  Her 1st pregnancy 12 months ago was complicated by pre-eclampsia and peripartum cardiomyopathy (LVEF 35%). Thankfully she delivered a healthy baby via C-section. She has no other past medical history and is currently on losartan 25 mg daily and metoprolol succinate 200 mg daily.  She has been asymptomatic. Which of the following statements is recommended to medically optimize Ms. Augustin prior to her 2nd pregnancy?

A

No medical optimization or preconception planning is needed as her 1st pregnancy resulted in a healthy infant.

B

Discontinue losartan and metoprolol with no other needed pregnancy planning 

C

Change her medication regimen, consider repeat TTE, and provide patient-centered counseling regarding risk of a future pregnancy

D

Continue losartan and metoprolol and advise against repeat pregnancy 

Explanation

The correct answer is C – change her medication regimen, consider repeat TTE, and provide patient-centered counseling regarding risk of a future pregnancy.

Heart failure may complicate pregnancy either secondary to an existing pre-pregnancy cardiomyopathy or as a result of peripartum cardiomyopathy. In women with history of heart failure or cardiomyopathy, including previous peripartum cardiomyopathy, patient-centered counseling regarding contraception and the risks of cardiovascular deterioration during pregnancy should be provided (Class I, LOE C-LD)

Peripartum cardiomyopathy (PPCM) is defined as systolic dysfunction, typically LVEF < 45%, often with LV dilation, occurring in late pregnancy or early postpartum with no other identifiable etiology. PPCM occurs worldwide, with the highest incidences in Haiti, Nigeria, and South Africa. Other clinical risk factors include maternal age > 30 years, African ancestry, multiparity, multigestation, preeclampsia/eclampsia, anemia, diabetes, obesity, and prolonged tocolysis.

 

The pathogenesis of peripartum cardiomyopathy is complex and it is likely a multifactorial process.  The combination of hemodynamic changes of pregnancy, inflammation of the myocardium, hormonal changes, genetic factors, and an autoimmune response have all been proposed as possible mechanisms and these may certainly be interrelated.

While pregnancy is generally well-tolerated in women with cardiomyopathy and NYHA class I status pre-pregnancy, clinical deterioration can occur and so counseling and shared decision-making are important. In fact, the ROPAC study of pregnancy outcomes for women with structural heart disease showed that women with pre-pregnancy or previous peripartum CM had the highest mortality rate at 2.4%. Subsequent pregnancies for women with previous peripartum cardiomyopathy have been associated with further decreases in LV function, maternal death, and adverse fetal outcomes. LVEF < 50% prior to a subsequent pregnancy is the strongest prognostic determinant.

Different strategies are needed to optimize the cardiovascular health of women with a prior history of PPCM before embarking on a subsequent pregnancy including pre-conception counseling regarding risk of subsequent pregnancies, pharmacologic strategies, and a multi-disciplinary approach to expectant management.

Pre-conception counseling: can utilize cardiovascular risk tools including ZAHARA I and CARPREG II scores (which predict outcomes during pregnancy in women with prior heart disease) and obtain a baseline TTE prior to conception to inform shared decision making.  

Pharmacologic strategies: in women with HF or cardiomyopathy who are

pregnant or currently planning for pregnancy, ACEi, ARB, ARNi, MRA, SGLT2i, ivabradine, and vericiguat should not be administered because of significant risks of fetal harm (Class 3: Harm, LOE C-LD). Beta blockers (preferably metoprolol), hydralazine, and nitrates are considered acceptable during pregnancy, when guided by multidisciplinary shared decision-making regarding benefits and potential risks.  Diuretic dosing should be discussed (if applicable) to minimize the risk of placental hypoperfusion. A repeat TTE should be performed 3 months following changes in heart failure medicine regimen. Of note, postpartum women who breastfeed can start an ACEi (enalapril or captopril are preferred), and metoprolol remains the preferred beta blocker.

Multidisciplinary care may include consultations with genetics, gynecology, and maternal-fetal medicine teams, as appropriate to the outcome of shared decision making. During pregnancy, for women with decompensated HF or evidence of hemodynamic instability antepartum, delivery planning will include obstetrics and maternal-fetal medicine, cardiac anesthesia, cardiology, and neonatology teams.

Therefore, answer choice C is correct because pre-conception counseling is essential to guide pertinent discussions on risk stratification prior to subsequent pregnancies. Additionally, her medications need to be modified by discontinuing her ARB prior to conception.

Choice A is incorrect because she is high risk for worsening cardiomyopathy and repeat preeclampsia in her next pregnancy.

Choice B is incorrect because shared decision making and risk stratification prior to 2nd pregnancy are essential.

Choice D is incorrect because, although she is at high risk for complications including worsening cardiomyopathy, preeclampsia/eclampsia, and neonatal demise, repeat pregnancy is not absolutely contraindicated and should be an informed decision after appropriate education within the construct of a multidisciplinary team.

See Heart Failure and Pregnancy Infographic created by Dr. Alaa Diab, CardioNerds Academy Chief.

Main Takeaway

In summary, when a patient with history of peripartum cardiomyopathy is planning on a repeat pregnancy, patient-centered counseling regarding risks and management strategies should be provided with guidance from a multidisciplinary team and medications should be adjusted to balance GDMT for heart failure against risks to fetal development. 

Guideline Loc.

Section 11.3, Table 30

Mrs. Framingham is a 65-year-old woman who presents to her cardiologist’s office for stable angina and worsening dyspnea on minimal exertion. She has a history of non-insulin dependent type 2 diabetes mellitus and hypertension. She is taking metformin, linagliptin, lisinopril, and amlodipine. Blood pressure is 119/70 mmHg. Labs are notable for a hemoglobin of 14.2 mg/dL, iron of 18 mcg/dL, ferritin 150 ug/L, transferrin saturation 15%, and normal creatine kinase. An echocardiogram shows reduced left ventricular ejection fraction of 25%. Coronary angiography shows obstructive lesions involving the proximal left anterior descending, left circumflex, and right coronary arteries. In addition to optimizing GDMT, which of the following are recommendations for changes in management?

A

Anticoagulation, percutaneous revascularization, and IV iron

B

A change in her diabetic regimen, percutaneous revascularization, and PO iron

C

A change in her diabetic regimen, surgical revascularization, and IV iron

D

A change in her diabetic regimen, medical treatment alone for CAD, and PO iron

E

Anticoagulation and surgical revascularization

Explanation

The correct answer is C – a change in her diabetic regimen, surgical treatment and IV iron.

Multimorbidity is common in patients with heart failure. More than 85% of patients with HF also have at least 2 additional chronic conditions, of which the most common are hypertension, ischemic heart disease, diabetes, anemia, chronic kidney disease, morbid obesity, frailty, and malnutrition. These conditions can markedly impact patients’ tolerance to GDMT and can inform prognosis.

Not only was Mrs. F found with HFrEF (most likely due to ischemic cardiomyopathy), but she also suffers from severe multi-vessel coronary artery disease, hypertension, and non-insulin dependent type 2 diabetes mellitus.

In addition to starting optimized GDMT for HF, specific comorbidities in the heart failure patient warrant specific treatment strategies. Mrs. Framingham would benefit from a change in her diabetic regimen, namely switching from linagliptin to an SGLT2 inhibitor (e.g., empagliflozin, dapagliflozin). In patients with HF and type 2 diabetes, the

use of SGLT2i is recommended for the management of hyperglycemia and to reduce HF related morbidity and mortality (Class 1, LOE A).

Furthermore, as she has diabetes, symptomatic severe multi-vessel CAD, and LVEF≤35%, surgical revascularization with coronary artery bypass grafting is warranted to improve symptoms, cardiovascular hospitalizations, and long-term all-cause mortality (Class 1, LOE B-R). Given the severity of her coronary disease, presence of diabetes mellitus, and coronary anatomy suitable for bypass, percutaneous (i.e., PCI) or medical treatment alone are inappropriate (options B, D).

 

Although she does not have anemia, she may benefit from IV iron. IV iron supplementation has been shown in the FAIR-HF, IRONOUT HF, and AFFIRM-AHF trials to significantly improve NYHA functional class, 6-minute walk test, quality of life, and decrease hospitalizations for HF, independently of anemia. These effects were not seen with iron given orally (options B, D). Iron deficiency is usually defined as ferritin level <100 μg /L or 100 to 300 μg/L, if the transferrin saturation is <20%.  Therefore, in patients with HFrEF and iron deficiency with or without anemia, intravenous iron replacement is reasonable to improve functional status and QOL (Class 2a, LOE B-R).

 

Although HF is a pro-thrombotic state, anticoagulation is not warranted empirically in Mrs. F, who has no evidence of thrombus or high-risk features suggesting impending thrombus (options A, E).

Main Takeaway

In summary, multimorbidity is frequent in heart failure patients and treatment targeted to specific morbidities is warranted. In patients with heart failure and diabetes, an SGLT2 inhibitor should be part of the medication regimen. Intravenous iron supplementation should be considered in iron-deficient patients independent of anemia. In patients with heart failure with LVEF≤35% and severe coronary artery disease with suitable anatomy, coronary artery bypass grafting is recommended. 

Guideline Loc.

Section 10.1, Figure 14

Mrs. Hart is a 70-year-old woman hospitalized for a 2-week course of progressive exertional dyspnea, increasing peripheral edema, and mental status changes. She has a history of coronary artery disease, hypertension, and heart failure for which she takes aspirin, furosemide, carvedilol, lisinopril, and spironolactone. On physical exam, the patient is afebrile, BP is 80/60 mmHg, heart rate is 120 bpm, and respiratory rate is 28 breaths/min with O2 saturation of 92% breathing room air. She is sitting upright and is confused. Jugular venous pulsations are elevated. Cardiac exam reveals an S3 gallop. There is ascites and significant flank edema on abdominal exam. Her lower extremities have 2+ pitting edema to her knees and are cool to touch. Her labs are significant for an elevated serum Creatinine of 3.0 from a baseline of 1.0 mg/dL, lactate of 3.0 mmol/L, and liver enzyme elevation in the 300s U/L.

 

Which of the following is the most appropriate initial treatment?

A

Increase carvedilol

B

Start dobutamine

C

Increase lisinopril

D

Start nitroprusside

Explanation

The Correct answer is B – start dobutamine.

This patient with progressive congestive symptoms, mental status changes, and signs of hypoperfusion and end-organ dysfunction meets the clinical criteria of cardiogenic shock. The Class 1 recommendation is that in patients with cardiogenic shock, intravenous inotropic support should be used to maintain systemic perfusion and maintain end-organ performance (LOE B-NR). Their broad availability, ease of administration, and clinician familiarity favor such agents as first line when signs of hypoperfusion persist. Interestingly, despite their ubiquitous use for management of cardiogenic shock, there is a lack of robust evidence to suggest the clear benefit of one agent over another.  Therefore, the choice of a specific agent is guided by additional factors including vital signs, concurrent arrhythmias, and availability. For this patient, dobutamine is the only inotrope listed. Although she is tachycardic, her lack of arrhythmia makes dobutamine relatively lower risk and does not outweigh the potential benefits.

Choice A – Increase carvedilol – is not correct. Beta-blockers should be continued in HF hospitalization whenever possible; however, in a patient with low cardiac output and signs of shock, beta-blockers should be discontinued due to their negative inotropic effects.

Choice C – Increase lisinopril – is not correct. Afterload reduction is reasonable to decrease myocardial oxygen demand. However, given the hypotension and renal dysfunction, increasing lisinopril could be potentially dangerous by further exacerbating hypotension and renal dysfunction. Furthermore, given her tenuous hemodynamic status, it would be more beneficial to start an IV medication that is easier to monitor and rapidly titrate.

Choice D – Start nitroprusside – is not correct. Intravenous Vasodilators are helpful for improving cardiac output in high SVR states when the patient is normotensive or even hypertensive. However, this patient is HYPOtensive and so vasodilators should be held.

Main Takeaway

In patients with cardiogenic shock, intravenous inotropic support should be used to maintain systemic perfusion and preserve end-organ performance.

Guideline Loc.

Section 9.5

Mrs. Hart is a 63-year-old woman with a history of non-ischemic cardiomyopathy and heart failure with reduced ejection fraction (LVEF 20-25%) presenting with 5 days of worsening dyspnea and orthopnea. She takes carvedilol 12.5mg BID, sacubitril-valsartan 24-46mg BID, empagliflozin 10mg daily, and furosemide 40mg daily and reports that she has been able to take all her medications. What is the initial management for Mrs. H?

A

Assess her degree of congestion and hypoperfusion

B

Search for precipitating factors

C

Evaluate her overall trajectory

D

All of the above

E

None of the above

Explanation

The correct answer is D – all of the above.

 

Choice A is correct because in patients hospitalized with heart failure, the severity of congestion and adequacy of perfusion should be assessed to guide triage and initial therapy (Class 1, LOE C-LD). Congestion can be assessed by using the clinical exam to gauge right and left-sided filling pressures (e.g., elevated JVP, S3, edema) which are usually proportional in decompensation of chronic HF with low EF; however, up to 1 in 4 patients have a mismatch between right- and left-sided filling pressures. Hypoperfusion can be suspected from narrow pulse pressure and cool extremities, intolerance to neurohormonal antagonists, worsening renal function, altered mental status, and/or an elevated serum lactate. For more on the bedside evaluation of heart failure, enjoy Episode #142 – The Role of the Clinical Examination in Patients With Heart Failure – with Dr. Mark Drazner.

 

Choice B, searching for precipitating factors is also correct. In patients hospitalized with HF, the common precipitating factors and the overall patient trajectory should be assessed to guide appropriate therapy (Class 1, LOE C-LD). Common precipitating factors include ischemic and nonischemic causes, such as acute coronary syndromes, atrial fibrillation and other arrhythmias, uncontrolled HTN, other cardiac disease (e.g., endocarditis), acute infections, anemia, thyroid dysfunction, non-adherence to medications or new medications. When initial clinical assessment does not suggest congestion or hypoperfusion, symptoms of HF may be a result of transient ischemia, arrhythmias, or noncardiac disease such as chronic pulmonary disease or pneumonia, and more focused assessments may be warranted.

 

Lastly, Choice C, evaluation of a patient’s trajectory is correct as hospitalization for HF is a sentinel event that signals worse prognosis and provides key opportunities to redirect the disease trajectory – including establishment of optimal volume status before and after discharge. During the HF hospitalization, the approach to management should include and address precipitating factors, comorbidities, and previous limitations to ongoing disease management related to social determinants of health. The disease trajectory for patients hospitalized with reduced EF is markedly improved by optimization of recommended medical therapies, which should be initiated or increased toward target doses once the efficacy of diuresis has been shown.

Main Takeaway

In summary, when a patient is admitted for acute decompensated heart failure, initial management involves assessing the patient’s degree of congestion and hypoperfusion, identifying and addressing precipitating factors, and evaluating overall patient trajectory to guide appropriate triage and therapy.

Guideline Loc.

Section 9.1, Table 21

Mr. Shock is a 65-year-old man with a history of hypertension and non-ischemic cardiomyopathy (LVEF 25%) who is admitted with acute decompensated heart failure. He is currently being diuresed with a bumetanide drip, but is only making 20 cc/hour of urine. On exam, blood pressure is 85/68 mmHg and heart rate is 110 bpm. His JVP is at 12 cm and extremities are cool with thready pulses. Bloodwork is notable for a lactate of 3.5 mmol/L and creatinine of 2.5 mg/dL (baseline Cr 1.2 mg/dL). What is the most appropriate next step?

A

Augment diuresis with metolazone

B

Start sodium nitroprusside

C

Start dobutamine

D

Start oral metoprolol

E

None of the above

Explanation

The correct answer is C – start dobutamine.

 

In this scenario, the patient is in cardiogenic shock given hypotension and evidence of end-organ hypoperfusion on exam and labs. The patient’s cool extremities, low urine output, elevated lactate, and elevated creatinine all point towards hypoperfusion.

 

In patients with cardiogenic shock, intravenous inotropic support should be used to maintain systemic perfusion and preserve end-organ function (Class 1, LOE B-NR). Further, in patients with cardiogenic shock whose end-organ function cannot be maintained by pharmacologic means, temporary MCS is reasonable to support cardiac function (Class 2a, LOE B-NR).

 

The SCAI Cardiogenic Shock Criteria can be used to divide patients into stages. Stage A is a patient at risk for cardiogenic shock but currently not with any signs or symptoms, for example, a patient presenting with a myocardial infarction without present evidence of shock. Stage B is “pre-shock” – this may be a patient who has volume overload, tachycardia, and hypotension but does not have hypoperfusion based on exam and lab evaluation. Stage C is classic cardiogenic shock – the cold and wet profile. Bedside findings for Stage C shock include cool extremities, weak pulses, altered mental status, decreased urine output, and/or respiratory distress. Lab findings include impaired renal function, increased lactate, increased hepatic enzymes, and/or acidosis. Stage D is deteriorating with worsening hypotension and hypoperfusion with escalating use of pressors or mechanical circulatory support. Finally, stage E is extremis with refractory hypotension and hypoperfusion, with circulatory collapse. Our patient in the question stem is in SCAI stage C, or classic cardiogenic shock.

 

Choice A is incorrect. Augmenting diuresis with metolazone can be useful in a patient with diuretic resistance and decompensated heart failure. However, this patient is hypotensive and fits the wet and cool profile and will benefit from inotropic support to increase end organ perfusion.

 

Choice B is incorrect. Sodium nitroprusside can be used to increase cardiac output in cardiogenic shock and is particularly useful in patients with high systemic vascular resistance. Indeed, intravenous nitroglycerin and nitroprusside have a Class 2a indication (LOE B-NR) in patients who are admitted with decompensated HF without systemic hypotension as an adjuvant to diuretic therapy for relief of dyspnea. However, our patient is hypotensive and so vasodilators would not be appropriate at this time.

 

Choice C is incorrect. Metoprolol, a negative inotropic agent, should not be used in this patient with cardiogenic shock.

 

Relevant to this question is the use of invasive hemodynamic monitoring to guide therapy. The use a PA line has a Class 2b indication (LOE B-NR) in patients presenting with cardiogenic shock to define hemodynamic subsets and appropriate management strategies. Obtaining hemodynamic data via a PA line can also be particularly useful when escalating to mechanical circulatory support, when there is diagnostic uncertainty, or when a patient in shock is not responding to empiric initial shock measures. While the use of PA catheters has been controversial since the ESCAPE trial which showed no benefit in decompensated HF, the trial did not actually enroll patients with cardiogenic shock. Several observational studies have shown association between PA catheter use and improved outcomes in cardiogenic shock, particularly in conjunction with short-term MCS. PA catheters are a diagnostic tool and are best utilized when hemodynamic information can be translated into appropriate interventions, such as determining response to medical and MCS therapy, weaning off of MCS support, or uncovering right ventricular failure to guide appropriate therapy.

 

In the case of cardiogenic shock, studies have shown benefit with multidisciplinary teams of HF and critical care specialists, interventional cardiologists, and cardiac surgeons. Such teams should also be capable of providing appropriate palliative care. There is a Class 2a (LOE B-NR) recommendation for management of patients with cardiogenic shock by an experienced multidisciplinary team.

Main Takeaway

In summary, it is important to recognize cardiogenic shock early based on clinical criteria of hypotension and hypoperfusion and begin prompt initiation of IV inotropic agents such as dobutamine and/or MCS to optimize end-organ perfusion. When there is insufficient clinical improvement with initial measures, invasive hemodynamic assessment is recommended.

Guideline Loc.

Section 9.5

Tables 22-24

A 64-year-old woman with a history of chronic systolic heart failure secondary to NICM (LVEF 15-20%) s/p dual chamber ICD presents for routine follow-up. She reports several months of progressive fatigue, dyspnea, and peripheral edema. She has been hospitalized twice in the past year with acute decompensated heart failure. Efforts to optimize guideline directed medical therapy have been tempered by episodes of lightheadedness and hypotension. Her exam is notable for an elevated JVP, an S3 heart sound, and a III/VI holosystolic murmur best heard at the apex with radiation to the axilla. Labs show Na 130 mmol/L, Cr 1.8 mg/dL (from 1.1 mg/dL 6 months prior), and NT-proBNP 1,200 pg/mL. ECG in clinic shows sinus rhythm and a nonspecific IVCD with QRS 116 ms. Her most recent TTE shows biventricular dilation with LVEF 15-20%, moderate functional MR, moderate functional TR and estimated RVSP of 40mmHg. What is the most appropriate next step in management?

A

Refer to electrophysiology for upgrade to CRT-D

B

Increase sacubitril-valsartan dose

C

Refer for advanced therapies evaluation

D

Start treatment with milrinone infusion

Explanation

 

The correct answer is C – refer for advanced therapies evaluation.

Our patient has multiple signs and symptoms of advanced heart failure including NYHA Class III-IV functional status, persistently elevated natriuretic peptides, severely reduced LVEF, evidence of end organ dysfunction, multiple hospitalizations for ADHF, edema despite escalating doses of diuretics, and progressive intolerance to GDMT. Importantly, the 2018 European Society of Cardiology revised definition of advanced HF focuses on refractory symptoms rather than cardiac function and more clearly acknowledges that advanced HF can occur in patients without severely reduced LVEF, such as in those with isolated RV dysfunction, uncorrectable valvular or congenital heart disease, and in patients with preserved and mildly reduced LVEF.

In such patients with advanced heart failure, when consistent with the patient’s goals of care, timely referral for HF specialty care is recommended to review HF management and assess suitability for advanced HF therapies (eg, LVAD, cardiac transplantation, palliative care, and palliative inotropes) (Class I, LOE C-LD).

Clinical indicators of advanced heart failure should prompt a possible referral to an advanced HF specialist and can be remembered by the INEEDHELP acronym:

·       I – IV inotropes

·       N – NYHA IIIb-VI or persistently elevated natriuretic peptides

·       E – End-organ dysfunction

·       E – EF ≤ 35%

·       D – Defibrillator shocks

·       H – Hospitalizations > 1 in past year

·       E – Edema despite escalating diuretics

·       L – Low systolic blood pressure (≤90) or high heart rate

·       P – Prognostic medication; progressive intolerance or down-titration of GDMT

It would not be appropriate to refer to EP for CRT-D upgrade as this is a Class 3 recommendation (LOE B-R) in patients with QRS duration <120 ms for no benefit.

Increasing the dose of sacubitril-valsartan would not be appropriate in this setting as the patient would be likely unable to tolerate a higher dose given her complaints of lightheadedness and episodes of hypotension.

Initiating treatment with IV inotropes would not be appropriate in this setting. Although the use of IV inotropes is given a Class 1 recommendation (LOE B-NR) for the treatment of cardiogenic shock, the patient described in the question stem does not meet clinical criteria for cardiogenic shock.

Main Takeaway

Clinical indicators for advanced heart failure can be remembered by the I-Need-Help acronym, and there is a Class 1, LOE C recommendation for these patients to be referred to HF specialists for further management and assessment for advanced therapies, when consistent with the patient’s goals of care.

Guideline Loc.

Section 8.1

Tables 16-18

 

Stage D (Advanced) Heart Failure

Ms. Heffpefner is a 54-year-old woman who comes to your office for a routine visit. She does report increased fatigue and dyspnea on exertion without new orthopnea or extremity edema. She was previously diagnosed with type 2 diabetes, morbid obesity, obstructive sleep apnea, and TIA. She is currently prescribed metformin 1000mg twice daily, aspirin 81mg daily, rosuvastatin 40mg nightly, and furosemide 40mg daily. In clinic, her BP is 140/85 mmHg, HR is 110/min (rhythm irregularly irregular, found to be atrial fibrillation on ECG), and BMI is 43 kg/m2. Transthoracic echo shows an LVEF of 60%, moderate LV hypertrophy, moderate LA enlargement, and grade 2 diastolic dysfunction with no significant valvulopathy. What is the best next step?

A

Provide reassurance

B

Refer for gastric bypass

C

Refer for atrial fibrillation ablation

D

Start metoprolol and apixaban

Explanation

The correct answer is D – start metoprolol and apixaban.

Ms. Hefpeffner has a new diagnosis of atrial fibrillation (AF) and has a significantly elevated risk for embolic stroke based on her CHA2DS2-VASc score of 6 (hypertension, diabetes, heart failure, prior TIA, and female sex). The relationship between AF and HF is complex and the presence of either worsens the status of the other. Managing AF in patients with HFpEF can lead to symptom improvement (Class 2a, LOR C-EO). However, large, randomized trial data are unavailable to specifically guide therapy in patients with AF and HFpEF.

 

Generally, management of AF involves stroke prevention, rate and/or rhythm control, and lifestyle / risk-factor modification. With regards to stroke prevention, patients with chronic HF with permanent-persistent-paroxysmal AF and a CHA2DS2-VASc score of ≥2 (for men) and ≥3 (for women) should receive chronic anticoagulant therapy (Class 1, LOE A). When anticoagulation is used in chronic HF patients with AF, a DOAC is recommended over warfarin in eligible patients (Class 1, LOE A).

The decision for rate versus rhythm control should be individualized and reflects both patient symptoms and the likelihood of better ventricular function with sinus rhythm. For patients with HF and symptoms caused by AF, AF ablation is reasonable to improve symptoms and QOL (Class 2a, LOE B-R). However, referring for catheter ablation would be premature before first attempting rate control and instituting anticoagulation therapy.

 

Traditionally, beta-blockers and nondihydropyridine calcium channel blockers are used as first-line agents for rate control in AF. Interestingly, a small open-label trial, RATE-AF in elderly patients with AF and symptoms of HF (mostly with preserved LVEF), compared bisoprolol to digoxin. Although the primary endpoint of quality of life at 6 months was similar between the 2 groups, several secondary QOL endpoints, functional capacity, and reduction in NT-proBNP favored digoxin at 12 months, with similar rate reductions in both groups. More side effects (such as dizziness, lethargy, and hypotension) were seen with bisoprolol than with digoxin. However, digoxin has a narrow therapeutic window and needs to be monitored more closely.

Option A (provide reassurance) is inappropriate as this patient has heart failure with preserved EF, defined by signs and symptoms of HF in patients with an LVEF of 50% or more. Echocardiogram hints in this case include LV hypertrophy and diastolic dysfunction. Our patient also has comorbidities frequently associated with HFpEF such as hypertension, diabetes, OSA, and obesity. Other common comorbidities include CAD, CKD, and atrial arrhythmias. When diagnosing HFpEF, care must be taken to rule out mimicking conditions such as pulmonary hypertension or amyloidosis. A large portion of the management of HFpEF includes managing comorbid conditions such as hypertension, OSA, and atrial fibrillation. At this time, she is symptomatic with atrial fibrillation and rapid ventricular response, and warrants both rate control and stroke prophylaxis.

 

Although gastric bypass should be considered for patients with a BMI >35 kg/m2 with comorbidities (such as HTN or diabetes) and patients with a BMI > 40 kg/m2 independent of comorbid conditions, this is not the best next step at this time. First, she should receive anticoagulation to reduce the risk of stroke and achieve better control of her HR and BP.

 

Patients with HFpEF and hypertension should have medication titrated to attain blood pressure targets in accordance with published clinical practice guidelines to prevent morbidity (Class 1, LOE C-LD). Although the optimal BP goal and antihypertensive regimen in patient with HFpEF is not known, HFpEF trials so far have shown that RAAS antagonists including ACEi, ARB, MRA and possibly ARNi could be first-line agents to treat HTN in patients with HFpEF. Beta blockers may be used to treat hypertension in patients with a history of

MI, symptomatic CAD, or AF with rapid ventricular response. These effects need to be balanced with the potential contribution of chronotropic incompetence to exercise intolerance in some patients.

Main Takeaway

In patients with HFpEF, the diagnosis and management of comorbidities are very important, especially the treatment of HTN (Class 1, LOE C-LD) and AF (Class 2a, LOE C-EO).

Guideline Loc.

Section 7.7.1, Figure 12

Section 10.2

Mr. Flo Zin is a 64-year-old man who comes to discuss persistent lower extremity edema and dyspnea with mild exertion. He takes amlodipine for hypertension but has no other known comorbidities. In the clinic, his heart rate is 52 bpm and blood pressure is 120/70 mmHg. Physical exam reveals mildly elevated jugular venous pulsations and 1+ bilateral lower extremity edema. Labs show an unremarkable CBC, normal renal function and electrolytes, a Hb A1c of 6.1%, and an NT-proBNP of 750 (no prior baseline available). On echocardiogram, his LVEF is 44% and nuclear stress testing was negative for inducible ischemia. What is the best next step in management?

A

Add furosemide BID and daily metolazone

B

Start empagliflozin and furosemide as needed

C

Start metoprolol succinate

D

No change to medical therapy

Explanation

The correct answer is B – start empagliflozin and furosemide as needed.

The patient described here has heart failure with mildly reduced EF (HFmrEF), given LVEF in the range of 41-49%.

In patients with HF who have fluid retention, diuretics are recommended to relieve congestion, improve symptoms, and prevent worsening HF (Class 1, LOE B-NR). For patients with HF and congestive symptoms, addition of a thiazide (eg, metolazone) to treatment with a loop diuretic should be reserved for patients who do not respond to moderate or high-dose loop diuretics to minimize electrolyte abnormalities (Class 1, LOE B-NR). Therefore, option A is not correct as he is only mildly congested on examination, and likely would not require such aggressive decongestive therapy, particularly with normal renal function. Adding a thiazide diuretic without first optimizing loop diuretic dosing would be premature.

The EMPEROR-Preserved trial showed a significant benefit of the SGLT2i, empagliflozin, in patients with symptomatic HF, with LVEF >40% and elevated natriuretic peptides. The 21% reduction in the primary composite endpoint of time to HF hospitalization or cardiovascular death was driven mostly by a significant 29% reduction in time to HF hospitalization, with no benefit on all-cause mortality. Empagliflozin also resulted in a significant reduction in total HF hospitalizations, decrease in the slope of the eGFR decline, and a modest improvement in QOL at 52 weeks. Of note, the benefit was similar irrespective of the presence or absence of diabetes at baseline. In a subgroup of 1983 patients with LVEF 41% to 49% in EMPEROR-Preserved, empagliflozin, an SGLT2i, reduced the risk of the primary composite endpoint of cardiovascular death or hospitalization for HF. Therefore, in patients with HFmrEF, SGLT2i can be beneficial in decreasing HF hospitalizations and cardiovascular mortality (Class 2a, LOE B-R). Furthermore, by inhibiting glucose reabsorption in the kidney, they have a diuretic effect which may help ease congestion and limit loop diuretic dosing. SGLT2i are beneficial to the vast majority of cardiovascular patients but are contraindicated in patients with type 1 diabetes or prior episodes of diabetic ketoacidosis as they may cause euglycemic DKA.

Option C is incorrect. Among patients with current or previous symptomatic HFmrEF (LVEF, 41%–49%), use of evidence-based beta blockers for HFrEF, ARNi, ACEi, or ARB, and MRAs may be considered to reduce the risk of HF hospitalization and cardiovascular mortality, particularly among patients with LVEF on the lower end of this spectrum (Class 2b, LOE B-NR). However, the patient’s heart rate is already low and so initiating a beta blocker would be inappropriate. Switching his calcium channel blocker to ARNi may be considered.

Option D is not correct as we can help counsel him on lifestyle and medication changes which can relieve his symptoms and reduce his risk of HF hospitalizations and mortality.

Main Takeaway

In patients with HFmrEF, diuretics are useful for decongestion and symptomatic improvement (Class 1) and there is a role for GDMT including SGLT2i (Class 2a) and BB, ARNI, ACEi/ARB, MRA (Class 2b).

Guideline Loc.

Section 7.6.1, Figure 11

Ms. Flo Zinn is a 60-year-old woman seen in cardiology clinic for follow up of her chronic HFrEF management. She has a history of stable coronary artery disease, hypertension, hypothyroidism, and recurrent urinary tract infections. She does not have a history of diabetes and recent hemoglobin A1c is 5.0%. Her current medications include carvedilol, sacubitril-valsartan, eplerenone, and atorvastatin. Her friend was recently placed on an SGLT2 inhibitor and asks if she should be considered for one as well. Which of the following is the most important consideration when deciding to start this patient on an SGLT2 inhibitor?

A

The patient does not have a history of type 2 diabetes and so does not qualify for SGLT2 inhibitor therapy

B

While SGLT2 inhibitors improve hospitalization rates for HFrEF, there is no evidence that they improve cardiovascular mortality

C

Patients taking SGLT2 inhibitors tend to suffer a more rapid decline in renal function than patients not taking SGLT2 inhibitor therapy

D

Patients may be at a higher risk for genitourinary infections if an SGLT2 inhibitor is started

Explanation

 

The correct answer is D – SGLT2 inhibitors have been associated with increased risk of genitourinary infections.

Sodium-glucose co-transporter protein 2 (SGLT2) inhibitors have gathered a lot of press recently as the new kid on the block with respect to heart failure management. While they were initially developed as antihyperglycemic medications for treating diabetes, early cardiovascular outcomes trials showed reduced rates of heart failure hospitalization amongst study participants independent of glucose-lowering effects and irrespective of baseline heart failure status – only 10-14% of patients carried a heart failure diagnosis at baseline. This prompted trials to study the effects of SGLT2 inhibitors in patients with symptomatic chronic HFrEF who were already on guideline directed medical therapy irrespective of the presence of type 2 diabetes mellitus. The DAPA-HF and EMPEROR-Reduced trials showed that dapagliflozin and empagliflozin, respectively, both conferred statistically significant improvements in a composite of heart failure hospitalizations and cardiovascular death (Option B). Most interestingly, these effects were seen irrespective of diabetes history. In light of these findings, the 2022 HF guidelines recommend SGLT2 inhibitors in patients with chronic, symptomatic HFrEF with or without diabetes to reduce hospitalization for HF and cardiovascular mortality (Class I, LOE A).

The benefits of SGLT2 inhibitors extend beyond cardiovascular health. Analyses of the DAPA-HF and EMPEROR-Reduced trials showed that patients receiving SGLT2 inhibitor therapy had fewer serious renal outcomes and slower rates of decline in eGFR than patients in the control groups.

As with all medications, though, SGLT2 inhibitors must be used with an awareness of some potentially serious side effects. SGLT2 inhibitors have been associated with higher rates of genitourinary infections, potentially related to the increased glycosuria associated with sodium-glucose co-transporter 2 inhibition. Trials have shown a 2 to 4-fold increased risk of vulvovaginal candidiasis for patients on SGLT2is compared to placebo. SGLT2 inhibitor use has also been associated with bacterial urinary tract infections, Fournier’s gangrene, and euglycemic ketoacidosis.

Main Takeaway

SGLT2 inhibitors are now a class I recommendation for patients with chronic symptomatic HFrEF regardless of whether or not they have diabetes. Although SGLT2i increased risk for genital infections, they were otherwise well tolerated in the trials.

Guideline Loc.

Section 7.3.4

Ms. Valarie Sartan is a 55-year-old woman with a history of HFrEF (EF 35%) and well controlled, non-insulin dependent diabetes mellitus who presents to heart failure clinic for routine follow up. She is currently being treated with metoprolol succinate 200mg daily, lisinopril 10mg daily, empagliflozin 10mg daily, and spironolactone 50mg daily. She notes stable dyspnea with moderate exertion, making it difficult to do her yardwork. On exam she is well appearing, and blood pressure is 115/70 mmHg with normal jugular venous pulsations and trace bilateral lower extremity edema. On labs, her potassium is 4.0 mmol/L and creatinine is 0.7 mg/dL with an eGFR > 60 mL/min/1.73m2. Which of the following options would be the most appropriate next step in heart failure therapy? 

A 

Increase lisinopril to 40mg daily 

B 

Increase spironolactone to 100mg daily 

C 

Add sacubitril-valsartan to her regimen 

Discontinue lisinopril and start sacubitril-valsartan in 36 hours 

E 

No change 

Explanation  

The correct answer is D – transitioning from an ACEi to an ARNi is the most appropriate next step in management.  

The renin-angiotensin aldosterone system (RAAS) is upregulated in patients with chronic heart failure with reduced ejection fraction (HFrEF). Blockade of the RAAS system with ACE inhibitors (ACEi), angiotensin receptor blockers (ARB), or angiotensin receptor neprilysin inhibitors (ARNi) have proven mortality benefit in these patients.  

The PARADIGM-HF trial compared sacubitril-valsartan (an ARNi) with enalapril in symptomatic patients with HFrEF. Patients receiving ARNi incurred a 20% relative risk reduction in the composite primary endpoint of cardiovascular death or heart failure hospitalization. Based on these results, the 2022 heart failure guidelines recommend replacing an ACEi or ARB for an ARNi in patients with chronic symptomatic HFrEF with NYHA class II or III symptoms to further reduce morbidity and mortality (Option D). This is a class I recommendation with level of evidence of B-R and is also of high economic value. Making no changes at this time would be inappropriate (Option E). 

While it would be reasonable to increase the dose of lisinopril to 40mg (Option A), this should be pursued only if ARNi therapy is not tolerated.  

Mineralocorticoid receptor antagonists (MRAs) have a class I (LOE A) recommendation in patients with HFrEF and NYHA class II to IV to reduce morbidity and mortality, provided that eGFR is >30 mL/min/1.73 m2 and serum potassium is <5.0 mEq/L, and there is careful monitoring of potassium, renal function, and diuretic dosing. However, the starting dose of spironolactone (or eplerenone) is 25 mg orally daily, increased to 50 mg daily orally after a month. Higher doses may be appropriate for other indications but are not advocated for HFrEF as the sole indication and so option B is incorrect.  

Guidance on starting an ARNi 

While switching from an ACEi to an ARNi, note that ARNi should not be administered concomitantly with ACEi or within 36 hours of the last dose of an ACEi (Class 3 for Harm, LOE B-R). This recommendation comes largely from studies of omapatrilat—a combination ACEi/neprilysin inhibitor. Patients receiving omapatrilat suffered significantly increased risk of angioedema thought secondary to dual suppression of both ACE and neprilysin leading to high concentrations of bradykinin. The current guidelines therefore recommend a washout period of at least 36 hours between the last ACEi dose and the first ARNi dose. If this patient were being transitioned from an ARB such as valsartan, then the first dose of ARNi could simply be given in lieu of the next anticipated dose of ARB.  

When initiating sacubitril-valsartan, it is important to monitor for signs of hypotension. With this patient’s blood pressure of 115/70 mmHg in clinic, she should have enough blood pressure room to tolerate the new medication; both PARADIGM-HF (ARNi vs ACEi in stable chronic HFrEF) and PIONEER-HF (ARNi vs ACEi in hospitalized patients with ADHF) excluded patients with SBP < 100 mmHg. That said, every patient responds differently, and anticipatory guidance should be given to anybody starting a new drug. In particular, Ms. H.F. should be counseled on symptoms that could reflect low blood pressure, such as lightheadedness or orthostatic syncope, asked to call her provider should she experience anything concerning. Laboratory follow-up should include renal function and potassium levels. ARNis should not be initiated on any patient with a history of angioedema (Class III for Harm, LOE C-LD). While this patient likely does not have this history since she is tolerating and ACEi, it is an important part of any CardioNerd’s checklist when reaching for RAAS inhibitors.  

Main Takeaway 

Patients with symptomatic HFrEF who are tolerating ACEi or ARB therapy should be transitioned to ARNi therapy to further reduce morbidity and mortality. 

Expert Suggestions 

  • Offer tips & tricks for initiating and monitoring ARNis in the outpatient setting.  
  • Discuss PIONEER-HF & initiation in the inpatient setting.  
  • Discuss ACEi & ARB combination therapy. 

Guideline Loc. 

Section 7.3.1  

Mr. D is a 50-year-old man who presented two months ago with palpations and new onset bilateral lower extremity swelling. Review of systems was negative for prior syncope. On transthoracic echocardiogram, he had an LVEF of 40% with moderate RV dilation and dysfunction. EKG showed inverted T-waves and low-amplitude signals just after the QRS in leads V1-V3. Ambulatory monitor revealed several episodes non-sustained ventricular tachycardia with a LBBB morphology.

He was initiated on GDMT and underwent genetic testing that revealed 2 desmosomal gene variants associated with arrhythmogenic right ventricular cardiomyopathy (ARVC).

Is the following statement true or false?

“ICD implantation is inappropriate at this time because his LVEF is >35%”

True

 

False

 

Explanation

This statement is False. ICD implantation is reasonable to decrease sudden death in patients with genetic arrhythmogenic cardiomyopathy with high-risk features of sudden death who have an LVEF ≤45% (Class 2a, LOE B-NR).

While the HF guidelines do not define high-risk features of sudden death, the 2019 HRS expert consensus statement on evaluation, risk stratification, and management of arrhythmogenic cardiomyopathy identify major and minor risk factors for ventricular arrhythmias as follows:

  • Major criteria: NSVT, inducibility of VT during EPS, LVEF ≤ 49%.
  • Minor criteria: male sex, >1000 premature ventricular contractions (PVCs)/24 hours, RV dysfunction, proband status, 2 or more desmosomal variants.

According to the HRS statement, high risk is defined as having either three major, two major and two minor, or one major and four minor risk factors for a class 2a recommendation for primary prevention ICD in this population (LOE B-NR).

Based on these criteria, our patient has 2 major risk factors (NSVT & LVEF ≤ 49%), and 3 minor risk factors (male sex, RV dysfunction, and 2 desmosomal variants) for ventricular arrhythmias. Therefore, ICD implantation for primary prevention of sudden cardiac death is reasonable.

Decisions around ICD implantation for primary prevention remain challenging and depend on estimated risk for SCD, co-morbidities, and patient preferences, and so should be guided by shared decision making weighing the possible benefits against the risks, especially in younger patients.

Main Takeaway

In patients with genetic arrhythmogenic cardiomyopathy with high-risk features of sudden death with LVEF ≤ 45%, implantation of ICD is reasonable.

Guideline Loc.

Section 7.4

Also: Section 3.10 from “Towbin, J. A., McKenna, W. J., Abrams, D. J., Ackerman, M. J., Calkins, H., Darrieux, F. C. C., Daubert, J. P., de Chillou, C., DePasquale, E. C., Desai, M. Y., Estes, N. A. M., Hua, W., Indik, J. H., Ingles, J., James, C. A., John, R. M., Judge, D. P., Keegan, R., Krahn, A. D., … Zareba, W. (2019). 2019 HRS expert consensus statement on evaluation, risk stratification, and management of arrhythmogenic cardiomyopathy. Heart Rhythm, 16(11), e301–e372. https://doi.org/10.1016/j.hrthm.2019.05.007”

 

Ms. L is a 65-year-old woman with nonischemic cardiomyopathy with a left ventricular ejection fraction (LVEF) of 35%, hypertension, and type 2 diabetes mellitus. She has been admitted to the hospital with decompensated heart failure (HF) twice in the last six months and admits that she struggles to understand how to take her medications and adjust her sodium intake to prevent this.  Which of the following interventions has the potential to decrease the risk of rehospitalization and/or improve mortality?

A

Access to a multidisciplinary team (physicians, nurses, pharmacists, social workers, care managers, etc) to assist with management of her HF

 

B

Engaging in a mobile app aimed at improving HF self-care

 

C

Vaccination against respiratory illnesses

 

D

A & C

 

 

The correct answer is D – both A (access to a multidisciplinary team) and C (vaccination against respiratory illness).

 

Choice A is correct. Multidisciplinary teams involving physicians, nurses, pharmacists, social workers, care managers, dieticians, and others, have been shown in multiple RCTs, metanalyses, and Cochrane reviews to both reduce hospital admissions and all-cause mortality. As such, it is a class I recommendation (LOE A) that patients with HF should receive care from multidisciplinary teams to facilitate the implementation of GDMT, address potential barriers to self-care, reduce the risk of subsequent rehospitalization for HF, and improve survival.

Choice B is incorrect.  Self-care in HF comprises treatment adherence and health maintenance behaviors. Patients with HF should learn to take medications as prescribed, restrict sodium intake, stay physically active, and get vaccinations. They also should understand how to monitor for signs and symptoms of worsening HF, and what to do in response to symptoms when they occur. Interventions focused on improving the self-care of HF patients significantly reduce hospitalizations and all-cause mortality as well as improve quality of life. Therefore, patients with HF should receive specific education and support to facilitate HF self-care in a multidisciplinary manner (Class I, LOE B-R). However, the method of delivery and education matters. Reinforcement with structured telephone support has been shown to be effective. In contrast the efficacy of mobile health-delivered educational interventions in improve self-care in patients with HF remains uncertain.

Choice C is correct. In patients with HF, vaccinating against respiratory illnesses is reasonable to reduce mortality (Class 2a, LOE B-NR). For example, administration of the influenza vaccine in HF patients has been shown to reduce all-cause mortality and hospitalizations.

Main Takeaway

Implementation of multidisciplinary care teams has been proven to reduce rehospitalization and mortality in HF patients. While education on self-care of HF patients is important, not all delivery methods have been shown to be effective.

Guideline Loc.

Section 7.1

Mr. Stevens is a 55-year-old man who presents with progressively worsening dyspnea on exertion for the past 2 weeks. He has associated paroxysmal nocturnal dyspnea, intermittent exertional chest pressure, and bilateral lower extremity edema. Otherwise, Mr. Stevens does not have any medical history and does not take any medications. 

Which of the following will be helpful for diagnosis at this time?

A

 Detailed history and physical examination

B

 Chest x-ray

C

 Blood workup including CBC, CMP, NT proBNP

D

 12-lead ECG

E

All of the above

 

 

The correct answer is E – All of the above. 

Mr. Stevens presents with signs and symptoms of volume overload concerning for new onset heart failure. The history and physical exam remain the cornerstone in the assessment of patients with HF. Not only is the H&P valuable for identifying the presence of heart failure but also may provide hints about the degree of congestion, underlying etiology, and alternative diagnoses. As such H&P earns a Class 1 indication for a variety of reasons in patients with heart failure:

1.       Vital signs and evidence of clinical congestion should be assessed at each encounter to guide overall management, including adjustment of diuretics and other medications (Class 1, LOE B-NR)

2.       Clinical factors indicating the presence of advanced HF should be sought via the history and physical examination (Class 1, LOE B-NR)

3.       A 3-generation family history should be obtained or updated when assessing the cause of the cardiomyopathy to identify possible inherited disease (Class 1, LOE B-NR)

4.       A thorough history and physical examination should direct diagnostic strategies to uncover specific causes that may warrant disease-specific management (Class 1, LOE B-NR)

5.       A thorough history and physical examination should be obtained and performed to identify cardiac and noncardiac disorders, lifestyle and behavioral factors, and social determinants of health that might cause or accelerate the development or progression of HF (Class 1, LOE C-EO)

Building on the H&P, laboratory evaluation provides important information about comorbidities, suitability for and adverse effects of treatments, potential causes or confounders of HF, severity and prognosis of HF, and more. As such, for patients who are diagnosed with HF, laboratory evaluation should include complete blood count, urinalysis, serum electrolytes, blood urea nitrogen, serum creatinine, glucose, lipid profile, liver function tests, iron studies, and thyroid-stimulating hormone to optimize management (Class 1, LOE C-EO). In addition, the specific cause of HF should be explored using additional laboratory testing for appropriate management (LOE 1, LOE B-NR). In patients presenting with dyspnea such as Mr. Stevens, measurement of B-type natriuretic peptide (BNP) or N-terminal prohormone of B-type natriuretic peptide (NT-proBNP) is useful to support a diagnosis or exclusion of HF (Class 1, LOE A); and in those with chronic HF, measurements of BNP or NT-proBNP levels are recommended for risk stratification (Class 1, LOE A).

In addition to bloodwork, electrocardiography is part of the routine evaluation of a patient with HF and provides important information on rhythm, heart rate, QRS morphology and duration, cause, and prognosis of HF. So for all patients with HF, a 12-lead ECG should be performed at the initial encounter to optimize management (Class 1, LOE C-EO).

Imaging is essential in the diagnosis and management of heart failure. In patients with suspected or new-onset HF, or those presenting with acute decompensated

HF, a chest x-ray should be performed to assess heart size and pulmonary congestion and to detect alternative cardiac, pulmonary, and other diseases that may cause or contribute to the patient’s symptoms (Class 1, LOE C-LD). Additionally, in those with suspected or newly diagnosed HF, transthoracic echocardiography (TTE) should be performed during the initial evaluation to assess cardiac structure and function (Class 1, LOE C-LD); and when echocardiography is inadequate, alternative imaging (e.g.,  cardiac

magnetic resonance [CMR], cardiac computed tomography [CT], radionuclide imaging) is recommended for assessment of LVEF (Class 1, LOE C-LD).

Main Takeaway 

In patients who present with signs and symptoms of volume overload concerning for new-onset heart failure, it is essential to rule out non-cardiac causes and assess for specific underlying causes of heart failure by using detailed history and physical examination. Once heart failure diagnosis is established, further workup with laboratory testing, ECG, and non-invasive cardiac imaging is warranted to investigate the etiology of heart failure and guide further management. Special attention should be given to detection of signs and symptoms suggesting an advanced stage of disease.

Guideline Loc. 

Section 4.1

Which of the following is/are true about heart failure epidemiology?

A

Although the absolute number of patients with HF has partly grown, the incidence of HF has decreased

B

Non-Hispanic Black patients have the highest death rate per capita resulting from HF

C

In patients with established HF, non-Hispanic Black patients have a higher HF hospitalization rate compared with non-Hispanic White patients

D

In patients with established HF, non-Hispanic Black patients have a lower death rate compared with non-Hispanic White patients

E

All of the above

Explanation 

 

The correct answer is “E – all of the above.”

 

Although the absolute number of patients with HF has partly grown as a result of the increasing number of older adults, the incidence of HF has decreased. There is decreasing incidence of HFrEF and increasing incidence of HFpEF. The health and socioeconomic burden of HF is growing. Beginning in 2012, the age-adjusted death-rate per capita for HF increased for the first time in the US. HF hospitalizations have also been increasing in the US. In 2017, there were 1.2 million HF hospitalizations in the US among 924,000 patients with HF, a 26% increase compared with 2013.

 

Non-Hispanic Black patients have the highest death rate per capita. A report examining the US population found the age-adjusted mortality rate for HF to be 92 per 100,000 individuals for non-Hispanic Black patients, 87 per 100,000 for non-Hispanic White patients, and 53 per 100,000 for Hispanic patients.

 

Among patients with established HF, non-Hispanic Black patients experienced a higher rate of HF hospitalization and a lower rate of death than non-Hispanic White patients with HF.Hispanic patients with HF have been found to have similar or higher HF hospitalization rates and similar or lower mortality rates compared with non-Hispanic White patients.

Asian/Pacific Islander patients with HF have had a similar rate of hospitalization as non-Hispanic White patients but a lower death rate.

 

These racial and ethnic disparities warrant studies and health policy changes to address health inequity.

Main Takeaway

Racial and ethnic disparities in death resulting from HF persist, with non-Hispanic Black patients having the highest death rate per capita, and a higher rate of HF hospitalization. Further clinical studies and health policy changes are needed to address these inequalities.

Guideline Loc.

Section 3.1

A 67-year-old man with a past medical history of type 2 diabetes mellitus, hypertension, and active tobacco smoking presents to the emergency room with substernal chest pain for the past 5 hours. An electrocardiogram reveals ST segment elevations in the anterior precordial leads and he is transferred emergently to the catheterization laboratory. Coronary angiography reveals 100% occlusion of the proximal left anterior descending artery, and he is successfully treated with a drug eluting stent resulting in TIMI 3 coronary flow. Following his procedure, a transthoracic echocardiogram is performed which reveals a left ventricular ejection fraction of 35% with a hypokinetic anterior wall. Which of the following medications would be the best choice to prevent the incidence of heart failure and reduce mortality?

A

Lisinopril

B

Diltiazem

C

Carvedilol

D

Sacubitril-valsartan

E

Both A and C

The correct answer is E – both lisinopril and carvedilol are appropriate to reduce the incidence of heart failure and mortality.

Evidence-based beta-blockers and ACE inhibitors both have Class 1 recommendations in patients with a recent myocardial infarction and left ventricular ejection fraction ≤ 40% to reduce the incidence of heart failure and to reduce mortality. Multiple randomized controlled trials have investigated both medications in the post myocardial infarction setting and demonstrated improved ventricular remodeling as well as benefits for mortality and development of incident heart failure.

At this time, there is not sufficient evidence to recommend ARNi over ACEi for patients with reduced LVEF following acute MI. The PARADISE-MI trial randomized a total of 5,661 patients with myocardial infarction complicated by a reduced LVEF, pulmonary congestion, or both to receive either sacubitril-valsartan (97-103mg twice daily) or ramipril (5mg twice daily). After a median follow up time of 22 months, there was no statistically significant difference in the primary outcome of cardiovascular death or incident heart failure. At this time, ARNi have not been included in the guidelines for this specific population.

Diltiazem is a non-dihydropyridine calcium channel blocker, a family of drugs with negative inotropic effects and which may be harmful in patients with depressed LVEF (Class 3: Harm, LOE C-LD).

Main Takeaway: 

For patients with recent myocardial infarction and reduced left ventricular function both beta blockers and ACEi have Class 1 recommendations to reduce the incidence of heart failure and decrease mortality.

Guideline Location:

Section 6.1

A 23-year-old man presents to his primary care physician for an annual visit. His father was diagnosed with idiopathic cardiomyopathy at 40 years of age. His blood pressure in clinic is 146/90 mmHg. He is a personal trainer and exercises daily, including both weightlifting and cardio. He denies any anabolic steroid use. He is an active tobacco smoker, approximately ½ pack per day. Review of systems is negative for symptoms. What stage of heart failure most appropriately describes his current status?

A

Stage A

B

Stage B

C

Stage C

D

Stage D

E

None of the above

Listen to this podcast episode here. 

The correct answer is A – Stage A of heart failure.

Overall, the ACC/AHA stages of HF were designed to emphasize the development and progression of disease. More advanced stages and progression are associated with reduced survival.

Stage A HF is where patients are “at risk for HF”, but without current or previous symptoms or signs of HF, and without structural/functional heart disease or abnormal biomarkers. At-risk patients include those with hypertension, cardiovascular disease, diabetes, obesity, exposure to cardiotoxic agents, genetic variant for cardiomyopathy, or family history of cardiomyopathy.

Stage B HF is the “pre-heart failure” stage where patients are without current or previous symptoms or signs of HF but do have at least one of the following:

Structural heart disease (i.e., reduced left or right ventricular systolic function, ventricular hypertrophy, chamber enlargement, wall motion abnormalities, and valvular heart disease)

  • Evidence of increased filling pressures
  • Risk factors and increased natriuretic peptide levels or persistently elevated cardiac troponin in the absence of an alternate diagnosis

Stage C HF indicates symptomatic heart failure where patients have current or previous symptoms or signs of HF.

Stage D HF indicates advanced heart failure with marked HF symptoms that interfere with daily life and with recurrent hospitalizations despite attempts to optimize guideline-directed medical therapy.

Therapeutic interventions in each stage aim to modify risk factors (Stage A), treat risk and structural heart disease to prevent HF (stage B), and reduce symptoms, morbidity, and mortality (stages C and D).

Given this patient’s family and social histories, along with the clinical finding of elevated blood pressure, he is best classified as having Stage A, or at risk for HF. Were he to have signs of cardiac abnormalities on chest X-ray, ECG, biomarkers, or other testing, he would then be classified as having Stage B, or pre-heart failure.

Main Takeaway:

It is important to identify patients who are at risk for heart failure (Stage A HF) early to modify risk factors and prevent disease progression.

Guideline location:

Section 2.1, Figure 1, Table 3